Chap 20 Flashcards Preview

Renal II Midterm > Chap 20 > Flashcards

Flashcards in Chap 20 Deck (206)
Loading flashcards...
1
Q

_ is incompatible with life found in stillborn infants. Due to this defect, amniotic fluid is not made, and the baby gets crushed consistent with potter’s sequence.

A

Bilateral agenesis

2
Q

_ is compatible with compatible with life without other abnormalities. The good kidney will become enlarged and hypertrophy, and some develop sclerosis and renal failure.

A

Unilateral agenesis

3
Q

how is a congenital hypoplastic kidney distinguished from an acquired atrophic kidney

A

A true hypoplastic kidney has reduced lobes and pyramids (

4
Q

Ectopic kidneys usually develop where?

A

Pelvic brim or within pelvis

5
Q

What are some common complications of ectopic kidneys?

A

The ectopic kidneys are usually small and symptomatic. They may cause torsion or obstruction of ureter, predisposing for inferior (pyelonephritis)

6
Q

A shorsehoe kidney is continuous across the anterior of the aorta and inferior vena cava and get caught by what structure?

A

Inferior mesenteric artery.

7
Q

Ectopic kidneys most common is fusion of which pole, upper or lower?

A

Lower pole (90% of cases)

8
Q

Horseshoe kidneys have an increased risk of

A

stone formation

9
Q

Cystic renal dysplasis is an abnormality in _

A

metanephrotic differentiation

10
Q

What is cystic renal dysplasia characterized by?

A

Histologically by the persistence in the kidney of abnormal structures (cartilage, undifferentiated mesenchyme, and immature collecting ducts) and by abnormal lobe organization. Large cysts surrounded by mesenchyme

11
Q

On gross examination, how does cystic renal dysplasia appear?

A
  • uni or bi lateral cysts, with extremely enlarged kidneys
12
Q

A 60 year old woman who has been on dialysis for number of years presents to the ER with flank pain and renal insufficiency. Pt stated she saw blood in her urine earlier during the day. On PE an abdominal mass was palpated. You suspect that patient has 1. The major complication of this disease is 2.

A
  1. Acquired (dialysis associated) cystic disease

2. Renal cell carcinoma

13
Q

Adult form of polycystic kidney disease has what kind of inheritance?

A

Autosomal dominant

14
Q

what is the pathogenesis of polycystic kidney disease?

A

Polycystic kidney disease genes (PKD1 and PKD2) on chromosome 16 and 14, respectively, are implicated in the generation of cyst via polycycstin protein abnormality leading to proliferation of different regions of the tubules

15
Q

Morphologically, this disease shows enlarged kidneys reaching enormous sizes (4kg/kidney). The surface appears entirely cystic though histology reveals functional parenchyma and arise from different regions of the tubules with cysts expressing variable epithelia.

A

Autosomal dominant polycystic kidney disease

16
Q

Patients presents with flank pain and renal insufficiency. she also stated that she has seen blood in her urine. On PE a large abdominal mass was palpable. On further examination abnormalities on PKD1 and PKD2 were noted and confimed autosomal dominant polycystic kidney disease. Extrarenal cyst is most often found where? and what is a common cause of death in these pts?

A

40% occur in the liver from biliary epithelium and are asymptomatic. Some occur in lungs or spleen; mitral valve prolapse in heart.

10% of deaths occur from intracranial berry aneurysms.

17
Q

what is the Gene implicated in autosomal recessive polycystic kidney disease and what does it encode and what’s messed up due to the mutation?

A

PHKD1 which codes for fibrocystin expressed in adult/fetal kidneys liver/pancreas. Mutation screws up collecting tubule+ biliary epithelium differentiation.

18
Q

Describe how the cysts appear in autosomal recessive PKD.

A
  • enlarged, smooth surfaced, tiny elongated cysts along the interior perpendicular to the cortical surface. Lined by cuboidal cells cuz all cysts come from collecting tubules.
19
Q

What is the prognosis of autosomal recessive PKD?

A

highly fatal in infancy, obvious renal failure in all cases.

20
Q

Those infants who survive autosomal recessive PKD, will develop fibrosis of what other organ?

A

Liver, along with hypertension and splenomegaly.

21
Q

On radiology, simple localized renal cysts are often mistaken for tumors. Explain how they appear.

A

On radiology, a pts scan showed small cortical translucent, grey, glistening, that was lined by a single layered membrane.

22
Q

What possible symptoms do pts with simple renal cysts present with?

A

Usually pain and distention if the kidney bleeds into the cysts.

23
Q

Medullary sponge kidney is an innocuous medullary cystic disease that is found incidentally on radiograph. These pts have increased risk of _

A

Stones, hematuria, infections

24
Q

A. Juvenile nephronophthisis - Medullary cystic disease complex (malicious medullary cystic disease) is implicated due to abnormality in what gene? what protein does that gene make?
B. Adult form of nephronopthisis-Medullary cystic disease complex: what gene is implicated?

A

A. NPH gene which makes nephrocystin –> shrunken kidney

B. MCKD1

25
Q

Nephronophthisis-medullary cystic disease complex induces cysts formation in what location?

A

Corticomedullary junction and interstitial fibrosis.

26
Q

A 10 year old boy presents to the clinic with polyuria and polydypsia, urine was noticeably dilute. Radiograph was unremarkable. Family history is positive for an uncle who had a similar case and required a kidney transplant within 10 years after diagnosis. Genetics testing showed NPH gene abnormality. Due to this disease, cysts formation are localized to what location? and what is the prognosis?

A

Medullary cysts concentrated to the corticomedullary junction

27
Q

Cystic renal dysplasia has a good prognosis if _

A

unilateral and nephrectomy

28
Q

On IF glomerular damage due to Goodpasture’s appears as _

A

linear. Antibody-basement membrane due to Type II hypersensitivity.

29
Q

What is the hallmark of acute renal failure?

A

Azotemia, often with oliguria

30
Q

What is the common cause of prerenal azotemia?

A

Decreased blood flow to kidneys: Common cause of ARF.

31
Q

What is the BUN: Cr ratio in prerenal azotemia? how is Tubular function affect?

A

Greater than 15 (15 is normal) This increase is due to activation of RAAS. Tubular function is intact. FENa 500 indicating tubules retain function in concentrating urine

32
Q

What is the most common cause of postrenal azotemia?

A

Obstruction of the urinary tract downstream from kidney –> decreased outflow –> decreased GFR (due to increased back flow pressure) –> azotemia and oliguria.

33
Q

Explain early vs the chronic post renal azotemia

A

Early stage: Increased tubular pressure forces BUN into the blood thus BUN:Cr >15. Tubular function is normal.

Chronic stage: longstanding obstruction leads to tubular damage and thus tubular function is diminished thus BUN:Cr

34
Q

What are some intrarenal causes of azotemia?

A
  1. Acute Tubular necrosis

2. Acute interstitialnephritis

35
Q

_ is the most common cause of acute renal failure where necrotic cells plug tubules and thus GFR decreased leading to intrarenal azotemia. On urine sample brown granular casts can be seen.

A

Acute tubular necrosis

36
Q

Due to acute tubular necrosis, explain how BUN:Cr; fractional excretion of sodium, ability to concentrate urine changes.

A

BUN:Cr 2%

Osm

37
Q

Common causes of acute tubular necrosis include:

A
  1. Ischemia: PCT and TAL are susceptible. preceded by prerenal azotemia
  2. Nephrotoxic agents such as aminoglycodies, heavy metas, myoglobunuria, ethyelen glycol, radiocontrast dye and urate (PCT is esp susceptible
38
Q

Pt presents with sudden onset of oliguria with brown, granular casts. Elevated BUN and Creatinine. Hyperkalemia with acidosis is also noted. Pt medical history includes chemotherapy for metastatic carcinoma. Pt was worked up for acute renal failure. In this situation how long is oliguria likely to persist before recovery of her acute renal failure?

A

2-3 weeks. Pt presented with acute tubular necrosis due to urate possibly from her chemo. Renal tubular cells are stable cells and so it’ll take 2-3 weeks before those cells can reenter cell cycle and regenerate.

39
Q

65 yr old Patient presents with oliguria, fever, and a rash. An urine sample was taken which showed increased eosinophils. Medical history includes hypertension which was managed well with beta blockers until recently and her cardiologist added a diuretic which she started taking 2 weeks ago.
A. what is the best diagnosis?
B. If this disease was to progress, what structures are likely to be involved?

A

A. Acute interstitial nephritis, which is a drug-induced HSR of interstitial and tubules; results in ARF (intrarenal). Common causes are NSAIDS, PCN, and diuretics
B. Renal papillary necrosis.

40
Q

What are some causes of renal papillary necrosis? How do these pts usually present?

A

Chronic analgesic abuse, DM, Sickle cell trait or disease, severe acute pyelonephritis. Usually present with gross hematuria and flank pain

41
Q

Glomerular disorders with proteinuria greater than 3.5/day is grossly defined as a _ syndrome

A

Nephrotic syndrome

42
Q

what some common findings with nephrotic syndrome?

A
  • proteinuria (>3.5 g/day)
  • hypoalbuminemia –> edema
  • hypogammaglobuinemia –> risk of infection
  • Hypercoagulable state (loss of antithrombin III)
  • hyperlipidemia and hypercholesterolemia
43
Q

what are some common nephrotic syndrome

A
  1. Minimal change disease (MCD)
  2. Focal segmental glomerulonephrotic syndrome (FSGS)
  3. Membranous nephropathy
  4. membranoproliferative glomerulonephritis (type I and Type II)
44
Q

Focal segmental glomerulonephrotic syndrome (FSGS) is the most common cuase of nephrotic syndrome in what demographics?

A

Hispanics and african american

45
Q

FSGS is usually idiopathic, however, it has high association with _

A
  • HIV +
  • Heroin use
  • Sickle cell disease
46
Q

Explain how FSGS appear on H&E

A

Only a segment of the glomeruli will be involved and the rest of the glomeruli will be normal. And since it’s focal, it’ll only affect SOME of the glomeruli, NOT ALL

47
Q

A 47 HIV+ patient presents with presents with >3.5gm of protein in a 24 hour urine collection and lipiduria. His serum albumin is 2.9gm/dL. He has generalized edema. He fails to respond to steroid therapy. He has slight microscopic hematuria.
A. what is the most likely diagnosis.
B. what are you likely to see on EM?
C. what are you likely to see on IF?
D. Since pt did not respond to steroid, what is the next best option for treatment?

A

A. FSGS
B. Effacement of foot processes
C. NO immune complex deposits; negative IF
D. Renin-Angiotensin system inhibitors.

Side note: if you have a pt with nephrotic syndrome and did not respond to steroid therapy and progressed to chronic renal failure, pt most likely has FSGS.

48
Q

What is the most common cause of nephrotic syndrome in Caucasian adults?

A

membranous nephropathy

49
Q

What are usual causes of membranous nephropathy?

A
  • usually idiopathic; may be associated with HBV, HCV, solid tumors, SLE, drugs (NSAIDS, PCM)
50
Q

A 51 year old white female with lower extremity pitting edema and oliguria. He has 3.7gm of protein in a 24hr urine collection. A urine dipstick is positive for scant presence of microscopic red cells. Lab test shows hypoalbuminemia, and hyperlipidia although she has no history of cardiovascular disease. Past medical history includes SLE.
A. what is the most likely diagnosis?
B. What are you likely to find on H&E
C. what are you likely to find on IF (if present where would they located?)
D. What are likely to find on EM?

A

A. Membranous nephropathy.
B. Thick glomerular basement membrane
C. Due to immune complex deposition IF will appear granular
D. On EM you’ll find dome, spike, dome spike like pattern and this is due to podocytes responding to granular deposits on the basement membrane.

51
Q

What are the two mechanisms said to result in tubulointerstitial fibrosis?

A

A. Infarction to tubules, possibly from alteration of hemodynamics in FSGS conditions
B. Activation of tubule cells either from proteinuria or other cytokines. activated tubule cells express adhesion molecules and elute inflammatory cells that lead to fibrosis.

52
Q

Diffuse proliferation of glomerular cells with presence of leukocytes is defined as what disease?

A

Acute proliferative glomerulonephritis aka post streptococcal glomerulonephritis

53
Q

An 8 year old boy presents with malaise, fever, nausea, and hematuria. The boys mom indicated her son has just gotten over a strep throat but isn’t sure what is happening now. You suspect an complication of infection and so you do lab tests and one of the test shows elevated ASO tiers. In pathogenesis of kidney issues leading to hematuria,
A. antibody against what antigen results in immune complex formation and deposition?
B. What are you likely to see in H&E?
C. What are you likely to see on IF?
D. What are you likely to see on EM?
E. what is the prognosis with proper fluid/electrolyte therapy?
F. In rare cases, the kidney issue can progress to _

A

A. M protein antigen (ASO titer). after A beta-hemolytic pharyngitis or skin infection.
B. Enlarged and hypercellular glomeruli; tubules often contain red cells casts;
C. Granular deposits of IgM IgG, and C3
D. Discrete, amorphous, electron dense deposit on the subepithelial side (which is the immune complex), often termed “lumpy bumpy”
E. 95% recover well as Ag-Ab is cleared with fluid/electrolyte
F. 1% develop rapidly progressive glomerulonephritis. Prolonged and persistent proteinuria/abnormal GFR marks a poor prognosis

54
Q

45 year old female patient has suffered from hypertension with multifocal lung hemorrhage and renal disease for several years and control/management has been difficult. Now she has gross hematuria, oliguria and is rapidly progressing to anuria. her serum creatinine is significantly elevated, and red cells casts. Histology of her kidney showed crescents with proliferative parietal cells, macrophages, PMNs and fibrin strands between cells. IF showed granular pattern.
A. what is the diagnosis?
B. How does the kidney look grossly?
C. What is the best option of treatment in this case?

A

A. Type II RPGN (usually seen in postinfectious, SLE, or Henolch-Schloen)
B. Enlarged and pale kidney with cortical petchial hemorrhage.
C. Phasmapphoresis.

55
Q

What is the cause of Type I RPGN?

A

anti-GBM antibodies as in Goodpasture; anti-collgen type IV

56
Q

In Berger disease aka IgA nephropathy, IgA deposits are found where and what’s best mode for detection?

A

This is a primary renal disease (most common nephropathy worldwide) where IgA deposits are found in the mesangium detected by IF. Systemic disease can also cause IgA deposits as in henoch-Schonlein purpura)

57
Q

An 8 year old boy brought to the ER by his mother presents with episodic hematuria. Mom tells the doctor that her son was getting over a “cold”. He seemed well until recently when the boy told his mom his “pee appeared bright red”. On urine sample cast casts were seen.
A. what is the diagnosis?
B. Specifically what is being being deposited in the kidney that’s causing the hematuria? What part of the kidney are the deposits?
C. What are you likely to see on IF?
D. What are you likely to see on H&E?
E. what is the pathogenesis of the injury?
F. What is the outcome if left untreated?

A

A. IgA Nephropathy
B. IgA1 (a form of IgA) deposited in mesangium of glomeruli
C. On IF, Granular mesangial pattern is seen
D. Microscopically, you’ll see mesangial proliferation or overt crescentic glomerulonephritis
E. After an upper respiratory infection/mucosal infection, IgA and IgA-complexes are formed that circulate in blood. some IgA become trapped in glomerulus leading to complement activation and glomerular injury
F. If left untreated, progresses to eventual renal failure.

58
Q

A 17 year old boy with a history of HBV infection, presents to the ER with severe proteinuria, slight hematuria, and pitting edema. On Lab, hypocomplementemia was indicated. On H&E of the kidney, thick capillary with tram-like appearance was seen.
A. What is the diagnosis?
B. What is the pathogenesis?
C. what specific kidney structures are involved?
D. what are you likely to see on IF
E. What are you likely to see on EM?
F. does does the two types of this disease differ?
G. Which has the worst prognosis, type I or II?

A

A. membranoproliferative glomerulonephritis Type I
B. Deposition of immune complex in subendothelial cells and mesangial deposits of C3 and IgG or IgM
C. immune complex deposition with subendothelial and mesangial deposits of C3 and IgG or IgM
D. IgG, IgM and C3 in a granular pattern pushed to the periphery
E. Subendothelial deposits (capillary lumen side, with RBCs)
F. Type I: subendothelial; associated with HBV and HCV, mostly a nephritic syndrome. Type II: Intramembranous; associated with C3 npehritic factor, an autoantibody that stablizes the C3 convertase, can be nephritic or nephrotic.
G. Type II, no effective treatment exists.

59
Q

Uncontrolled diabetes can lead to nephrotic syndrome.
A. Explain how
B. what you’re likely to see on H&E that is pathognomonic for this disorder.
C. What tx do these pts usually do well on?

A

High serum glucose leads to nonenzymatic glycosylation of vascular basement membrane. this is the first change that’s seen. This leads to BM becoming leaky and so protein leaks out leading to hyaline arteriosclerosis –> thickening of the blood vessels walls –> decreased caliber of the lumen (preferentially in the efferent arteriole). This builds up back pressure in the glomeruli –> hyperfiltration which begins first by microalbuminuria, eventually sclerosis of the mesengium leading to nephrotic syndrome.
B. On H&E you’ll see sclerosis of mesangium and formation of Kimmelsteil-Wilson nodules.
C. ACE inhibitors which works to slow the progression of hyperfiltration

60
Q

Explain the pathogenesis of how a nephrotic syndrome develops.

A
  1. An initial event causes derangement of glomerular cpaillaries with increased permeability to protein with resultant proteinuria.
  2. Albumin leaks out along with proteinuria leading to decreased colloid pressure
  3. Edema: periorbital or peripheral
  4. Hyperlipidemia/cholesteremia
61
Q

What is the most common nephrotic syndrome in adults?

A

membranous glomerulonephropathy

62
Q

Does membranous glomerulonephropathy respond well to Corticosteroids therapy?

A

No

63
Q

membranous glomerulonephropahty is a chronic immune complex deposition disease that can be idiopathic, however is associated nonidiopathic cuases such as _

A

LSE, drugs (PCM), tumors, metals (drugs/mercury), or infections (HCV, HBV, shistosomiasis, malaria)

64
Q

What is the most common cause of nephrotic syndrome in children?

A

minimal change disease.

65
Q

A 4 year old child presents with 4g of protein reflected in a 24 hour urine collection. He has periorbital edema.
A. How will corticosteroid therapy affect his condition?
B. What are you likely to see on EM?
c. how does the glomeruli appear on light microscopy?
D.

A

A. Responds very well to steroid therapy
B. Effaced foot processes, vacules, and fused podoyctes, which are actually just flattened epithelium
C. On LM glomerulus appear normal

66
Q

A 7 year old child presents with 3.6gm of protein in a 24 hour urine collection and lipiduria. His serum albumin is 2.9 mg/dL. he has generalized edema. Doc suspecting that it is minimal change disease, gives him steroid, however child does not improve.
A. What is the likely diagnosis?
B. what are you likely to find on biopsy?
C. What are you likely to find on IF?
D. What are you likely to see on EM?
E. In what racial / pt population demographics is this disease most common?

A

A. FSGS
B. Only a segment of the glomeruli will be involved and the rest of the glomeruli will be normal. and since its focal, it’ll only affect some of the glomeruli, not all.
C. IF is negative
D. EM will show affected areas with mesangial sclerosis and foot process fusion (effacement of the foot process)
E. Hispanics and African Americans. Also seen in patients who have HIV, are heroin user, have sickle cell disease.

67
Q

FSGS due to HIV has morphological distinctions. What are they?

A

Collapsing glomerulosclerosis where there are complete glomerular collapse and sclerosis with focal segmental changes of FSGS.

68
Q

Is alport’s syndrome nephritic or nephrotic?

A

Nephritic

69
Q

What is the inheritance pattern of Alport’s syndrome?

A

X-linked recessive

70
Q

Explain the pathogenesis of Alports

A

X linked disorder involving collagen formation. Defective glomerular basement membrane as a result of a defect in type IV collagen synthesis. There is a lack of alpha chains, particularly alpha 5 chain of collagen type IV

71
Q

A 5 year old boy presents with the doctor with visual problems. It was also noted that boy has diminished hearing. Upon workup, microscopic hematuria was detected on his urine sample.
A. what is the most likely diagnosis?
B. What are you likely to see on EM?
C. Overt kidney failure is likely to occur between what age?

A

A. Alport syndrome
B. alternating thickening and thinning glomerular basement membrane with lamination of the lamina densa.
C. 20-50 years.

72
Q

A 5 year old boy friends to his doc for a regular checkup. On UA, slight hematuria was noted. Everything else otherwise was normal. Further testing showed a defect in Type IV collagen formation.
A. what is the most likely diagnosis?
B. WHat are you likely to find on EM?
C. What is the progoniss?

A

A. Thin basement membrane disease
B. thining of glomerular basement membrane, with NO IgA immune deposition in the mesangium. (thus not IgA nephropathy).
C. Prognosis is excellent with maintenance of normal renal function throughout life.

73
Q

The common glomerular lesions associated with systemic disease include _

A
  1. Lupus
  2. Henoch-Schonlein Purpura
  3. Amyoidosis (associated with MM)
  4. Fibrillary glomerulonephritis (like amylodoisis except IgG and C3 on IF)
  5. Essential Mixed cryoglobulinemia (associated with HepC)
  6. Plasma Cell dyscarias (like amyloidosis and MM, light chain or monoclonal immune deposits)
  7. bacterial endocarditis
  8. Diabetic glomerulosclerosis/nepropathy
74
Q

In Lupus associated gomerulus lesion where do the antibody-antigen complex deposit?

A

glomerular filtration barrier. macrophages activation leads to injury and eventual fibrosis of the glomerulus and even some vasculitidies. Hematuria, acute nephritis, nephrotic syndrome and hypertension and eventually renal failure.

75
Q

Pt presents with puritic skin rahses on extensor surfaces of arms and legs. On examination the skin lesions contained necrotizing vasculitis within microhemorrhage. Further workup shoed IgA deposits on renal mesangium, some C3.
A. what is the most likely diagnosis?
B. What other kidney disease it this most associated with?

A

A. Henoch-Schonlein purpura

B. IgA nephropathy (Berger’s syndrome)

76
Q

Amyloidosis associated glomerular lesion, light chain deposits are located where?
B. what are you likely to find on UA?

A

large blood flows: glomerular capilaries and mesangium. In EM you’d see massive baement membrane without any deposits.
B. heavy proteinueira (large Uprotein/Ucreatinien ratio) with only trace protein on dipstick.

77
Q

56 year old patient presents to the ER complaining of oliguria. On Lab and UA microalbuminuria proteinuria were present. Morphologicaly, there were capillary basement membrane thickening, concomitantly with tubular basement membrane thickening. There were also diffuse mesangial sclerosis and nodular glomerulosclerosis known as Kimmelsteil-Wilson disease. what is the most likely diagnosis?

A

Diabetic glomerulosclerosis/nephropathy.

78
Q

Acute tubular necrosis is diagnostic with an urine sample that contains _

A

muddy-brown/granular casts on urine microscopy. aka renal failure casts

79
Q

Pyelonephritis is defined as

A

infection of the ureters and renal pelvis resulting in tubules, interstitium, and pelvis being affected by infection

80
Q

Commonly pyelonephritis is caused by ascending infection from a lower urinary tract infection. what are the most common causative agents of such infections which can lead to pyelonephritis?

A

E coli, proteus, Klebsiella, enterobacter, and staph

81
Q

Vesiculoreteal reflux is implicated in ascending infection leading to pyelonephritis, how?

A

Vesicolouretal reflux allows bacteria to gain access to ureters thereafter an easy route to ascend to the kidneys.

82
Q

39 yr old woman presents with severe malaise, and a high grade fever. she also complained of dysuria/frequency with sudden onset of pain in the costovertebral angle (Flank pain). Urine culture was positive for growth, and UA was positive for white cell casts.
A. what is the diagnosis?
B. Morphologiclaly what are you likely to find?
C. Start the patient on treatment and does well and recovers. What are you likely to find on morphology weeks later?

A

A. Acute pyelonephritis
B. Hallmark finding: patchy interstitial supprative inflammation, intratubular WBC aggregates (which will be seen in UA as WBC casts) and tubular necrosis. Affects upper and lower poles more than middle. Papillary necrosis is seen in diabetics with distal pyramid necrosis
C. fibrosis of the underlying renal pelvis and calyces.

83
Q

Tubulointerstial inflammation causing discrete, corticomedullary scars overlying dilated, blunted and deformed calyces are characteristics of what? What would be a poor prognostic finding?

A

A. Chronic pyelonephritis due to recurrent ascending infections.
B. Proteinuria

84
Q

A 60-year-old man with a history of osteoarthritis has been awaiting hip replacement surgery for 3 years. During his annual physical, he reports that no amount of analgesic can relieve his constant pain and NSAIDS. Patient now presents to the ER with a high grade fever and rash for the past day. Upon work up, oliguria and increased serum Creatinine is noted suggestive of acute renal failure. A renal biopsy shows eosinophilic infiltration and areas of healing and scarring. What is the most likely explanation for this patient’s acute renal failure?

A

Tubulointerstitial nephritis caused by use of use of NSAIDS. Drugs act as a hapten until concentrated in the tubules for excretion in urine whereby activation of IgE and T/B plasma cells in the localized area.

85
Q

A 60-year-old man with a history of osteoarthritis has been awaiting hip replacement surgery for 3 years. During his annual physical, he reports that no amount of analgesic can relieve his constant pain. Laboratory results reveal that his renal function has deteriorated when compared to his last office visit 2 years ago. Serum creatinine is 2.0 mg/dL and urinalysis shows 1+ proteinuria. There are no abnormalities seen on microscopy of the urine. A renal biopsy shows eosinophilic infiltration and diffuse parenchymal inflammation. What is the most likely explanation for this patient’s deterioration in renal function?

B. In this setting, damage to what structures occur first?

C. these patients are at increased risk for developing what neoplasm?

A

Toxic tubulointerstial nephritis aka Analgesic nephropathy. The question stem describes a patient who is overusing analgesics. In this scenario, the most likely explanation for deterioration in renal function is NSAID-induced toxic tubulointerstitial nephritis (TIN).

B. papillary necrosis, followed by tubulointerstittial nephritis.

C. Transitional cell carcinoma of the renal pelvis

86
Q

A 77 year old male with difficulty to control hypertension and hyperlipidemia presents to his doctor with complaining of oliguria. The doc suspects benign nephrosclerosis, however, the doc does a through work up because these pts are at increased risk of what kidney issues?
B. On morphology what are you likely to find?

A

Chronic renal failure.
B. sclerosis from medial and intimal thickening of arteries that are hyalinized with typical arteriolar changes. Overall the kidneys are normal to small with a leathery appearance in gross.

87
Q

A 65 year poorly controlled hypertensive pt presents with oliguria, papilledema, and slight retinopathy. UA showed proteinuria, hematuria. Lab showed slightly increased serum creatinine. Morphologically fibrinoid necrosis and onion skinning of basement membrane was noted.
A. what is the diagnosis?
B. What is the pathogenesis of the morphological changes?

A

A. malignant hypertension nephrosclerosis
B. Extreme BP –> endothelial cell damage, increased permeability, platelet activation, fibrin extravasation, necrosis, all summing up into fibrinoid necrosis. Kidney becomes ischemic and RAAS is activated which makes the HTN worse and cycle repeats.

88
Q

Unilateral renal artery stenosis is relatively uncommon and can result from: A.
B. These patients do very well with _

A

A. cardiovascular changes (plaques) that lead to decreased RBF elaborating ANGII via RAS
B. ACEI or ARB and surgery is definitive therapy

89
Q

What is the pentad associated with Thrombotic thrombocytopenia purpura TTP?

A

thrombocytopenia, microangiopathic hemolytic anemia, thrombosis.

90
Q

Hemolytic uremic syndrome (HUS) is similar to TPP but without _

A

neurologic symptoms but with renal failure.

91
Q

Classic (childhood) HUS is commonly cuased by _. How do these kids usually present?

A

E.coli hamburgers (0157:H7) containing shigga-like toxin. Toxin cuaes denudation of epithelium and decreased NO synthesis. Sudden onset of hemtemesis and melena following flu-like prodrome. Oliguria/hematuria/hemolytic anemia may follow. Managed by dialysis

92
Q

Adult HUS are usually caused by secondary or systemic disease such as _

A

LSE/Antiphosphlipid antibodies = HUS without immune deposition

  • Infection
  • Complication of pregnancy
  • vascular renal disease (wegner’s sclerderma, HTN)
  • Chemo
93
Q

In familial HUS, the defect is in what protein?

A

Compliment regulatory proteins leading to complement C3 activation, thrombosis and capillary destruction results.

94
Q

Idiopathic HUS is linked to _ gene

A

platelet aggregation ADAMTS gene

95
Q

A pt with sickle cell anemia presents for his routine blood transfusion. On work up he was noted to have hematuria, dilute urine, and 2.5g proteinuria. On EM aggregates were found around the vasa recta which is the likely cause of his decrease concentrating ability leading to dilute urine. What is the best diagnosis?

A

Sickle cell nephropathy.

96
Q

A pt presents with dysuria, fever, chills and perianal pain. On PE, a rectal exam proved a tender and soft prostate.
A. what is the likely diagnosis?
B. What are you likely to find on prostatic secretion analysis?
C. What is the common cause?
D. With frequent episodes of this event, what is it likely to progress to and how would the patient present then?

A

A. Acute bacterial prostatitis
B. greater than 9 neutrophils per high power view
C. E.coli and sometimes Staph or Enteroccoccus from a lower UTI (cysttis)
D. With frequent bouts of UTI –> frequent prostatic infection–> Chronic bacterial prostatitis. Pt will present with dysuria, but Pelvic discomfort and lower back pain will also be prominent. On EM you’ll see plasma cells, lymphocytes and macrophages in chronic forms vs acute form which will only show neutorphils

97
Q

How is granulomatous prostatitis usually diagnosed and what does this disease usually mimic?

A

Diagnosed histologically which may be necrotizing or nonnecrotizing. Mimics prostatic carcinoma. Usually due secondary to surgical manipulation or stromal infiltration of prostatic fluid

98
Q

Explain the pathogenesis of BPH

A

with age, hormonal levels changes. Estrogen increase sensitivity for DHT receptors in prostate, testosterone really doesn’t play any role. DHT (from Leydig cell) has an increased selectivity on epithelial nuclear androgen receptors that induce glandular and stromal proliferation

99
Q

A 85 year old African American presents to his doctor’s office with increasing complaints of nocturia, urgency, hesitancy, inability to stop/dribbling. On a rectal exam a firm rubbery mass was palpated. he is otherwise healthy.
A. What is the likely diagnosis
B. Enlargement usually beings on what locations?
C. What is a possible treatment?

A

A. BPH
B. transitional and periurethral zones
C. 5-alpha-reductase (testosterone –> DHT) inhibitor or surgery (older tx)

100
Q

A 65 year old male presents to your Harlem, NY office complaining of nocturia, urgency, hesitancy, inability to stop/dribbling. Pt also complains of pain with urination multiple times in the past year. His BMI is 29. On a rectal exam, firm, discrete nodules were palpated. You talk with the patient about possible causes of this symptoms and decided to screen the patient for PSA and PAP level. Both PSA and PAP were elevated although percent of free PSA were relatively low. You decide to a biopsy of the mass which confirms the diagnosis.
A. what is the likely diagnosis?
B. On biopsy, how would this disease differ from BPH?
C. What is the significance of the doubling time for these cells?
D. Morphologically where are the cell growth usually found?
E. Presence of what is a precursor for malignancy in these type of tumors?
F. According to Gleanson Grading system, what score is considered a benign tumor?
G. At What stage would you likely to see this invade to seminal vesicle?

A

A. Prostatic adenocarcinoam
B. BPH is usually involving double layers. adenocarcinomas are single layer of epithelium
C. Doubling time is slow (2-4 years) thus has an excellent 10 year survival rate (depending on stage)
D. Found in peiropheral zones with only a single layer of epithelial cells. In BPH it’ll be double layer and found on the periurethral and transitional zones
E. Prostatic Intraepithelial neoplasia (PIN) is a precursor for malignancy
F. A score of 2 (1+1) would be considered benign while a score of 10 would be considered malignant (5+%)
G. Stage III Capsular involvement or invasion to seminal vesicle.

101
Q

An Urothelial carcinoma that has an association with smoking and old age is _

A

transitional cell carcinoma

102
Q

An urothelial carcinoma that has an association with schistosomiasis infection is _

A

squamous cell carcinoma

103
Q

Babies with a mass in the kidney is usually have what tumors? What if the baby also has an elevated levels of VMA?

A

Wilm’s tumor. With elevated VMA it’s neuroblastoma

104
Q

A 35 patient presents with renal colic and a sharp flank pain. UA was positive for slight hematuria. On imagining staghorn caliculi was present. What is the likely diagnosis?

A

Struvite stones ( magnesium ammonium phosphate slats). Caused by urea-splitting bacteria.

Note: staghorn caliculi is also seen in cystine stones, but those usually occur in children.

105
Q

What is the most common renal stones?

A

Calcium oxalate stones. (75%) associated with hypercalciuria w/wo hypercalcemia.

106
Q

Patients with gout, or any leukemias are at increased risk of what kidney stones?

A

Uric acid stones

107
Q

Which kidney stone is associated with genetic disease that prevent reabsopriton of protein from the lumen.

A

Cystiene stones.

108
Q

_ is a benign tumor that appears small, pale, yellow gray, well-circumscribed nodules within the renal cortex.

A

Renal papillary adenoma

109
Q

_ is a tiny nonmalignant, gray-white firm nodules found within the pyramids and is associated with tuberous sclerosis where the tumor is called an angiomyolipoma.

A

Renal fibroma or hamartoma

110
Q

_ is an epithelial tumor composed of large, eosinophilic cells with benign looking nuclei and monster nucleoli, and on EM massive amounts of mitochondria is seen.

A

Oncocytoma

111
Q

Pt presents with hematuria. On lab pt was found to have polycythemia. On genetic studies, VHL gene mutation was shown.
A. what is the diagnosis?
B. large clear cells form from epithelium of what strutures?
C. Unilateral or bilateral?
D. What color do the masses usually appear?

A

A. Clear cell carcinoma, a form of renal cell carcinoma.
B. Proximal tubular epithelium
C. Unilateral
D. yellow cuz of foam like fat cells

112
Q

A 60 year old female who has been on dialysis for 9 years awaiting a renal transplant, presents with hematuria. On lab pt was found to have polycythemia. On Ct a 3cm mass was found on the right kidney, and 2cm mass on the left kidney.
A. what is the likely diagnosis?
B. What is the cause?
C. malignant cells arise from what structures?

A

A. Papillary carcinoma
B. associated with dialysis-associated cystic disease
C. Cuboidal or columnar epithelium arranged in papillary pattern.

113
Q

_ is a form of renal cell carcinoma with an eosinopholic cytoplasm, perinuclear halo, localized to vasculature.

A

Chromophobe

114
Q

what is the triad of symptoms associated with renal cell carcinoma and which one is the most common?

A

Hematuria (most common and reliable), flank pain, and mass)

115
Q

What are some of the most common paraneoplastic syndrome associated with renal cell carcinoma?

A

Polycythemia (Epo), hypercalcemia (PTH), hypertension (Aldo, renin), Cushings (ACTH), and femininzation (androgens)

116
Q

A 2 year old child presents to ER with nausea and hematuria. The child also appeared hypertensive and has been for almost a year. On PE, a abdominal mass was palpable. CT genetic studies confirmed diagnosis.
A. What is the diagnosis?
B. What gene is implicated in the disease?
C. What is the prognosis?

A

A. Wilm’s tumor (nephroblastoma)
B. WT-1
C. Pronosis of a unilateral tumor is good (90% survival)

117
Q

A 33-year-old female presents with recent onset of painful urination, fever, and right flank pain. Urinary sediment analysis is positive for the presence of white blood cell casts and Gram-negative bacteria. She has not recently started any new medications. What is the most likely diagnosis in this patient?

  1. Pelvic Inflammatory Disease
  2. Acute Interstitial Nephritis
  3. Pyelonephritis
  4. Cystitis
  5. Appendicitis
A
  1. Pyelonephritis

This patient is suffering from acute pyelonephritis. In the setting of systemic febrile illness, white blood cell casts on urinary sediment analysis are pathognomonic for pyelonephritis.

Acute pyelonephritis is most common in pre-menopausal females and Esherichia coli is the most common causative organism. Vesicoureteral reflux with secondary ascending infection from the bladder is the mechanism by which bacteria gain exposure to the kidney and may result in acute pyelonephritis. Clinical clues to pyelonephritis include nausea/vomiting, fever, flank pain, costovertebral angle (CVA) tenderness, urinary frequency/urgency, and dysuria.

Colgan et al. describe the diagnostic approach to acute pyelonephritis. The diagnosis is often made by the history and physical examination. Fever is common, though may be absent early in the infection and nearly all patients have flank pain. Therefore, a lack of flank pain should raise suspicion for other potential etiologies.

Ramakrishnan et al. summarize the therapeutic approach to treating acute pyelonephritis. Uncomplicated pyelonephritis can be treated as an outpatient, often successfully with a fluoroquinolone. Other oral options include amoxicillin-clavulanate, cephalosporins and trimethoprim-sulfamethoxazole.

118
Q

A 34-year-old female presents to her primary care physician with complaints of fevers, nausea/vomiting, and severe left flank pain that has developed over the past several hours. She denies any prior episodes similar to her current presentation. Physical examination is significant for a body temperature of 39.1 C and costovertebral angle tenderness. A urinalysis and urine microscopy are ordered. Which of the following findings on kidney histology would be expected in this patient?

  1. Interstitial fibrosis and lymphocytic infiltrate
  2. Neutrophils filling the lumens of the renal tubules
  3. Thickening of the capillaries and glomerular basement membrane
  4. Scarring of the glomeruli
  5. Enlarged, hypercellular glomeruli with ‘wire-looping’ of capillaries
A
  1. Neutrophils filling the lumens of the renal tubules.

This patient’s presentation is most consistent with acute pyelonephritis. The histology of acute pyelonephritis consists of neutrophils filling the renal tubular lumens.

119
Q

A 22-year-old sexually active, otherwise healthy female presents to her primary care physician complaining of several days of dysuria, frequency, urgency, and suprapubic pain. She denies fever, flank pain, vaginal itching, or vaginal bleeding/discharge. Which organism is most likely responsible for this patient’s symptoms?

  1. Staphylococcus saprophyticus
  2. Chlamydia trachomatis
  3. Proteus mirabilis
  4. Klebsiella pneumoniae
  5. Escherichia coli
A
  1. E.coli

The patient in this vignette is exhibiting symptoms classic for a urinary tract infection, or cystitis. The most common cause of uncomplicated cystitis and pyelonephritis is E. coli.

120
Q

A female infant is born with a mutation in PKD1 on chromosome 16. An abdominal ultrasound performed shortly after birth would most likely reveal which of the following?

  1. Bilateral kidney enlargement
  2. Microscopic cysts
  3. Horseshoe kidney
  4. Adrenal atrophy
  5. Normal kidneys
A
  1. Normal kidneys

PKD1 is a mutation present in 90% of autosomal dominant polycystic kidney disease (formerly “adult” PKD). In infants with ADPKD, ultrasound will show normal kidneys. Although microscopic cysts are present at birth, they are too small to be detected on abdominal ultrasound.

121
Q

A 35-year-old male with a history of hypertension presents with hematuria and abdominal discomfort. Ultrasound and CT scan reveal large, bilateral cysts in all regions of the kidney. The patient’s disease is most commonly associated with:

  1. Aortic stenosis
  2. Berger’s disease
  3. Diabetes mellitus
  4. Berry aneurysm
  5. Henoch-Schonlein purpura
A
  1. Berry aneurysm

The patient’s symptoms and imaging findings are suggestive of adult polycystic kidney disease, or autosomal dominant polycystic kidney disease (ADPKD). ADPKD is associated with Berry aneurysms in the circle of Willis in 10-15% of patients.

122
Q

A 10-year-old Caucasian female with Turner’s syndrome underwent an abdominal imaging study and was discovered that the poles of her kidneys were fused inferiorly. Normal ascension of kidney during embryological development would be prevented by which of the following anatomical structures?

  1. Inferior vena cava
  2. Superior mesenteric artery
  3. Inferior mesenteric artery
  4. Celiac artery
  5. Splenic artery
A
  1. IMA

The patient described in the question stem has a horseshoe kidney, the most common congenital kidney disorder. In horseshoe kidney, the isthmus of the kidney gets trapped behind the inferior mesenteric artery as the kidneys ascend during embryonic life.

123
Q

A 40-year-old male presents to the emergency room following a motorcycle accident. His blood pressure on arrival is 70/50 mmHg and his heart rate is 130 bpm. During hospitalization, he developed oliguria and has urine studies shown in Figure A. He is eventually discharged from the hospital with restored renal function. Which of the following was responsible for this patient’s kidney problems?

  1. Acute pyelonephritis
  2. Diabetic glomerulopathy
  3. Rapidly progressive glomerulonephritis
  4. Acute tubular necrosis
  5. Membranous glomerulonephritis
A
  1. Acute tubular necrosis

This patient’s renal failure was due to ischemic acute tubular necrosis, secondary to hypovolemia. Acute tubular necrosis classically results in oliguria, “muddy brown” pigmented renal tubular cell casts as seen in Figure A, hyperkalemia, and anion gap metabolic acidosis.

124
Q

A 62-year-old man presents to the emergency room with an acute myocardial infarction. Twenty-four hours after admission to the cardiac intensive care unit, he develops oliguria. Laboratory tests show that his serum BUN is 59 mg/dL and his serum creatinine is 6.2 mg/dL. Renal biopsy reveals necrosis of the proximal tubules and thick ascending limb of Henle’s loop. Which of the following would you most likely observe on a microscopic examination of this patient’s urine?

  1. White blood cell casts
  2. Fatty casts
  3. Muddy brown casts
  4. Hyaline casts
  5. Broad waxy casts
A
  1. muddy brown cast

This patient’s presentation is consistent with acute tubular necrosis (ATN). Muddy brown casts are part of the classic presentation of ATN.

125
Q

A 48-year-old woman is admitted to the hospital with sepsis and treated with gentamicin. One week after her admission, she develops oliguria and her urine shows muddy brown casts on light microscopy. Days later, her renal function begins to recover, but she complains of weakness and develops U waves on EKG as shown in Image A. Which laboratory abnormality would you most expect to see in this patient?

  1. Hypocalcemia
  2. Hypokalemia
  3. Hyponatremia
  4. Hypoglycemia
  5. Hypermagnesemia
A
  1. Hypokalemia

This patient’s initial presentation is consistent with acute tubular necrosis (ATN) due to aminoglycoside toxicity. While hospitalized, she progresses from the acute phase of ATN to the recovery phase of ATN. During the recovery phase, she develops hypokalemia, which results in weakness and EKG changes.

126
Q

A 45-year-old female presents to the emergency department with gross hematuria and acute, colicky flank pain. She denies any previous episodes of hematuria. She reports taking high doses of acetaminophen and aspirin over several weeks due to persistent upper back pain. The patient’s blood pressure and temperature are normal, but she is found to have proteinuria. Physical examination is negative for palpable flank masses. Which of the following is the most likely diagnosis:

  1. Diffuse cortical necrosis
  2. Chronic pyelonephritis
  3. Autosomal dominant polycystic kidney disease
  4. Papillary necrosis
  5. Acute Nephrolithiasis
A
  1. Papillary necrosis

Acute onset of gross hematuria, colicky flank pain, and proteinuria are characteristic of renal papillary necrosis.

Answer 1: Diffuse cortical necrosis is marked by acute infarction of both kidneys. It is associated with obstetric catastrophes and septic shock.

127
Q

A previously healthy 9-year-old, Caucasian girl presents to your office with severe abdominal pain. Her mother also mentions that she has been urinating significantly less lately. History from the mother reveals that the girl suffers from acne vulgaris, mild scoliosis, and had a bout of diarrhea 3 days ago after a family barbecue. Lab work is done and is notable for a platelet count of 97,000 with a normal PT and PTT. The young girl appears dehydrated, yet her serum electrolyte levels are normal. What is the most likely etiology of this girl’s urinary symptoms?

  1. Hypothalamic dysfucntion
  2. Surreptitious laxative use
  3. Toxic shock syndrome
  4. Shiga toxin production from Shigella
  5. Shiga-like toxin production from EHEC
A

5: Shiga-like toxin production from EHEC

This patient’s presentation is consistent with hemolytic uremic syndrome (HUS). HUS is most frequently due to infection with Shiga-like toxin-producing Escherichia coli O157:H7 following ingestion of contaminated hamburger meat which likely occurred at the family barbecue mentioned in this question.

128
Q

A 49-year-old female with a history of alcoholism was found lying unconscious on a bench at a local park and rushed to the emergency department. Upon arrival, the patient regained consciousness and complained of intense bilateral flank pain and a recent decrease in urination. Urine microscopy demonstrated abundant square crystals of calcium oxalate that looked like “folded envelopes.” Which of the following findings is most likely to be seen in this patient:

  1. Metabolic alkalosis
  2. Anion gap metabolic acidosis
  3. Non-anion gap metabolic acidosis
  4. Respiratory acidosis
  5. Respiratory alkalosis
A
  1. Anion gap metabolic acidosis

This patient is suffering from acute renal failure secondary to ethylene glycol (antifreeze) ingestion. Ehtylene glycol ingestion produces anion gap metabolic acidosis.

129
Q

A 55-year-old man with a history of chronic glomerulonephritis due to IgA nephropathy presents to your office with bone pain. Which of the following laboratory findings would you most expect upon analysis of this patient’s serum?

  1. Increased PTH, decreased calcium, increased phosphate, decreased calcitriol
  2. Decreased PTH, increased calcium, increased phosphate, increased calcitriol
  3. Increased PTH, increased calcium, decreased phosphate, decreased calcitriol
  4. Decreased PTH, decreased calcium, increased phosphate, decreased calcitriol
  5. Normal PTH, normal calcium, normal phosphate, normal calcitriol
A
  1. Increased PTH, decreased calcium, increased phosphate, decreased calcitriol

This patient’s presentation is consistent with osteodystrophy secondary to chronic renal failure resulting in secondary hyperparathyroidism. This condition presents with increased PTH, decreased calcium, increased phosphate, and decreased calcitriol.

130
Q

A 65-year-old man with a longstanding history of poorly-controlled hypertension and no recent illness or cardiac disease presents to clinic complaining of chest pain that is worse on inspiration and relieved by sitting forward. Vital signs are stable. On exam, you detect a friction rub. Routine labs show K 7.5 and Cr 5.1. If this patient’s clinical signs and laboratory findings are indicative of a chronic condition, what would be expected on urinary analysis?

  1. Muddy brown granular casts
  2. Red blood cell casts
  3. White blood cell casts
  4. Fatty casts
  5. Broad and waxy casts
A
  1. Broad and waxy cast

This patient is presenting with uremic pericarditis secondary to renal failure. Broad and waxy casts are seen on urinary analysis in chronic renal failure.

131
Q

A 72-year-old female recently fractured her hip in a fall. She suffers from regular joint pain in her fingers, and hip X-rays reveal low bone mineral density. She has a history of diabetes mellitus and was diagnosed 2 years ago with end-stage renal disease. Serum phosphate levels are markedly elevated. Which of the following likely contributes to her orthopedic problems?

  1. Decreased serum aldosterone
  2. Increased serum parathyroid hormone
  3. Increased serum glucose
  4. Chronic hypertension
  5. Chronic hypovolemia
A
  1. Increased serum parathyroid hormone

The patient’s symptom profile suggests renal osteodystrophy, which is mostly likely due to secondary hyperparathyroidism

132
Q

Multiple patients present to your office with hematuria following an outbreak of Group A Streptococcus. Biopsy reveals that all of the patients have the same disease, characterized by large, hypercellular glomeruli with neutrophil infiltration. Which patient has the best prognosis?

  1. 65-year-old nulliparous woman
  2. 50-year-old man with a history of strep infection
  3. 8-year-old boy who undergoes no treatment
  4. 38-year-old man with sickle cell trait
  5. 18-year-old man treated with corticosteroids
A
  1. 8-year-old boy who undergoes no treatment

Age is the most important prognostic factor in post-streptococcal glomerulonephritis. Ninety-five percent of affected children recover completely, compared with 25% of adults over 60 years old.

133
Q

A 10-year-old boy presents to your office with cola-colored urine and periorbital edema. His mother is extremely concerned, especially given that her son has been entirely healthy except for a sore throat a few weeks ago. Which of the following would you be LEAST likely to observe on a kidney biopsy of this patient?

  1. “Lumpy-bumpy” appearance on immunofluorescence
  2. Subepithelial electron dense deposits on electron microscopy
  3. Polyclonal IgG and C3 deposition on immunofluorescence
  4. Linear IgG deposition along the basement membrane on immunofluorescence
  5. Large, hypercellular glomeruli on light microscopy
A
  1. Linear IgG deposition along the basement membrane on immunofluorescence
    This patient’s clinical presentation of nephritic syndrome is consistent with post-streptococcal glomerulonephritis, also known as post-infectious GN. Linear IgG deposition along the basement membrane is characteristic of Goodpasture syndrome (anti-glomerular basement membrane disease) and would not likely be seen in this patient’s kidney biopsy.
134
Q

A 9-year-old Caucasian girl presents to your office with hematuria. An electron micrograph of her renal biopsy is shown below in Figure A. Which of the following is the most likely composition of the structures marked by the white arrows?

  1. Albumin
  2. Non-enzymatic glycosylation
  3. IgA
  4. IgG
  5. IgG, IgM, C3
A
  1. IgG, IgM, C3

The patient’s presentation and EM are consistent with post-streptococcal (postinfectious) glomerulonephritis. The white arrows demonstrate epimembranous immune complex deposits of IgG, IgM and C3.

135
Q

A 6-year-old boy presents to your office with hematuria. Two weeks ago the patient had symptoms of a sore throat and fever. Although physical exam is unremarkable, laboratory results show a decreased serum C3 level and an elevated anti-DNAse B titer. Which of the following would you most expect to see on renal biopsy?

  1. Large, hypercellular glomeruli on light microscopy
  2. Polyclonal IgA deposition on immunofluorescence
  3. Immune complex deposits with a “spike and dome” appearance on electron microscopy
  4. Wirelooping and hyaline thrombi on light microscopy
  5. Antibodies to GBM resulting in a linear immunofluorescence pattern
A
  1. Large, hypercellular glomeruli on light microscopy
    This patient’s presentation and laboratory results are consistent with post-streptococcal (post-infectious) glomerulonephritis (PSGN). On light microscopy, PSGN will show large, hypercellular glomeruli with neutrophil infiltration.
136
Q

A 21-year-old male presents to your office with hematuria 3 days after the onset of a productive cough and fever. Following renal biopsy, immunofluorescence shows granular IgA deposits in the glomerular mesangium. Which of the following do you suspect in this patient?

  1. Lipoid nephrosis
  2. Berger’s disease
  3. Poststreptococcal glomerulonephritis
  4. Systemic lupus erythematosus
  5. HIV infection
A
  1. Burger’s disease

Hematuria and immunofluorescence findings of IgA deposits in the mesangium suggest Berger’s disease (IgA glomerulonephropathy). Berger’s disease can occur concurrently or within several days of an infection.

137
Q

A 25-year-old male visits his primary care physician with complaints of hemoptysis and dysuria. Serum blood urea nitrogen and creatinine are elevated, blood pressure is 160/100 mm Hg, and urinalysis shows hematuria and RBC casts. A 24-hour urine excretion yields 1 gm/day protein. A kidney biopsy is obtained, and immunofluorescence shows linear IgG staining in the glomeruli. Which of the following antibodies is likely pathogenic for this patient’s disease?

  1. Anti-DNA antibody
  2. Anti-neutrophil cytoplasmic antibody (C-ANCA)
  3. Anti-neutrophil perinuclear antibody (P-ANCA)
  4. Anti-glomerular basement membrane antibody (Anti-GBM)
  5. Anti-phospholipid antibody
A
  1. Anti-glomerular basement membrane antibody (Anti-GBM)

The patient’s clinical picture, laboratory data, urinalysis, and light microscopy are consistent with Goodpasture syndrome. Goodpasture syndrome is a type II hypersensitivity reaction characterized by autoantibodies to the glomerular basement membrane (anti-GBM).

138
Q

Three weeks after recovering from pharyngitis, a nine-year-old girl presents with periorbital edema and dark brown urine. An image of the kidney biopsy is shown in Figure A. What is responsible for these changes?

  1. Proliferation of glomerulus parietal cells with fibrin and C3b accumulation
  2. IgA deposition and subsequent mesangial proliferation
  3. Neutrophil infiltration in response to immune complex deposition along the glomerulus basement membrane and mesangium
  4. Loss of negative charge on the glomerular basement membrane
  5. Widespread intermembraneous deposits due to an autoantibody against C3 convertase
A
  1. Neutrophil infiltration in response to immune complex deposition along the glomerulus basement membrane and mesangium

This young girl has acute poststreptococcal glomerulonephritis (PSGN) weeks after a streptococcal pharyngitis. Histopathology reveals immune complex deposition along the glomerular basement membrane (GBM) and mesangium and neutrophil infiltration.

139
Q

A 37-year-old man presents with significant hematuria and hemoptysis. The results of the immunoflouresence are shown in Figure A. What pathologic changes would be expected under light microscopy?

  1. “Wire looping” of the capillaries
  2. Hypercellular glomeruli
  3. Crescentic glomerulonephritis
  4. Focal proliferative glomerulonephritis
  5. Normal glomeruli
A
  1. Crescentic glomerulonephritis

The clinical scenario and attached image are consistent with Goodpasture syndrome, a type of rapidly progressive crescentic glomerulonephritis (RPGN).

140
Q

A 62-year-old man presents to his primary care doctor complaining of recent-onset hemoptysis. He has not had any fevers, night sweats, or weight loss. He recently traveled to Italy with his wife. He has a 5 pack-year smoking history. On review of systems, he reports that his urine has been “red-tinged” for several months. Urinalysis reveals the findings shown in Figure A. He is referred to a nephrologist and undergoes a renal biopsy. Immunofluorescence staining of the biopsy is shown in Figure B. Which of the following is the most likely underlying pathogenesis of his disease?

  1. Type III hypersensitivity reaction
  2. Antibodies to collagen type IV
  3. Presence of C-ANCA
  4. Mycobacterial infection
  5. Metastatic malignancy
A
  1. Antibodies to collagen type IV

This patient has pulmonary hemorrhage and nephritic syndrome. His renal biopsy shows linear deposition of antibodies along the basement membrane in the glomerulus, a finding characteristic of Goodpasture disease.

141
Q

A 5-year-old female presents to your office with periorbital edema and proteinuria. Her mother reports that she “just got over a cold” last week. On examination, you note a pulse rate of 70/minute and a blood pressure of 95/53. Which protein(s) would likely be found in large amounts in this patient’s urine?

  1. IgG
  2. IgA
  3. Albumin
  4. Albumin and IgG
  5. Albumin, IgG, and IgA
A
  1. Albumin
    This patient’s presentation is consistent with minimal change disease (MCD), the most common nephrotic syndrome in children. In MCD, T-cell cytokines cause the glomerular basement membrane to lose negative charge, resulting in SELECTIVE proteinuria in which albumin (but not immunoglobulins) is lost.
142
Q

An 11-year-old boy presents to your office with pitting edema and proteinuria exceeding 3.5g in 24 hours. You suspect that this patient has experienced a loss of polyanions in his glomerular basement membranes. Which of the following findings would confirm your diagnosis?

  1. WBC casts in the urine
  2. RBC casts in the urine
  3. Selective albuminuria
  4. Negatively birefringent crystals in the urine
  5. Bence-Jones proteinuria
A
  1. Selective albuminuria

The patient described in the question stem has minimal change disease (MCD), the most common cause of the nephrotic syndrome in children. MCD results in selective albuminuria.

143
Q

A 7-year-old male suffers from generalized edema. Urine protein excretion is 5.2 g over 24 hours and serum analysis reveals hyperlipidemia. The patient responds to treatment with prednisone, and, 8 weeks later, his urine does not contain measurable protein. If a kidney biopsy had been performed while the patient’s condition was pathologic, which of the following would you expect to find upon glomerular electron microscopy?

  1. Effacement of podocyte foot processes
  2. Subepithelial ‘spike and dome’ deposits
  3. Subepithelial humps
  4. Thin glomerular basement membrane
  5. Subendothelial thickening
A
  1. Effacement of podocyte foot processes
144
Q

A 57-year-old female visits her primary care physician with 2+ pitting edema in her legs. She takes no medications and does not use alcohol, tobacco, or illicit drugs. 4.5 grams of protein are collected during 24-hour urine excretion. A kidney biopsy is obtained. Examination with light microscopy shows diffuse thickening of the glomerular basement membrane. Electron microscopy shows subepithelial spike and dome deposits. Which of the following is the most likely diagnosis:

  1. Minimal change disease
  2. Postinfectious glomerulonephritis
  3. Focal segmental glomerulosclerosis
  4. Rapidly progressive glomerulonephritis
  5. Membranous glomerulopathy
A
  1. Membranous glomerulopathy

The patient’s symptoms and urine studies are indicative of nephrotic syndrome. Diffuse thickening of the glomerular basement membrane (GBM) and subepithelial spike and dome deposits are features of membranous glomerulopathy.

145
Q

A 6-year-old girl presents to your clinic two weeks after receiving a routine immunization in preparation for a trip overseas. Periorbital edema is present on exam and 24 hour urine collection shows excretion of 4.3 grams of protein/day. Which pathological change would likely be seen on microscopy?

  1. Linear IgG deposition on light microscopy
  2. IgA-immune complexes in the mesangium on electron microscopy
  3. “Tram-track” appearance on light microscopy
  4. Subepithelial deposits with “spike and dome” appearance on electron microscopy
  5. Podocyte effacement on electron microscopy
A
  1. Podocyte effacement on electron microscopy

The clinical scenario describes a child who is suffering from nephrotic syndrome secondary to minimal change disease (MCD). The pathognomic histologic finding is podocyte effacement on electron microscopy.

146
Q

A 55-year-old African American male presents to his primary care physician with complaints of persistent back pain and fatigue over 12 months. Physical examination reveals a blood pressure of 190/150 mm Hg, and laboratory tests reveal hyperlipidemia and a serum creatinine level of 3.0 mg/dL. 4.5 g of protein are excreted in the urine over 24 hours. Renal biopsy shows eosinophilic, acellular material in the glomerular tuft and capillary walls that display apple green-colored birefringence in polarized light upon Congo red tissue staining. The patient most likely suffers from which of the following:

  1. Membranous nephropathy
  2. Focal segmental glomerular sclerosis
  3. Drug-induced acute tubular necrosis
  4. Multiple myeloma
  5. Malignant hypertension
A
  1. MM

The patient’s clinical findings suggest amyloidosis (due to multiple myeloma), which can lead to nephrotic syndrome.

147
Q

A 67-year-old man presents to your office with a chief complaint of constipation and many other perturbing minor medical concerns. He reports tiring easily, which he attributes to old age and years of persistent pain in his back and ribs. A complete blood count shows low hemoglobin and elevated serum creatinine. A peripheral blood smear shows stacks of red blood cells among other findings, and serum electropheresis reveals an abnormal concentration of protein resulting in a spike. Which of the following additional findings would you expect to see in this patient?

  1. Early satiety and splenomegaly
  2. Smudge cells on peripheral smear
  3. An elevated PSA and a nodular prostate
  4. Bence-Jones proteins in the urine
  5. No additional findings - normal aging explains symptoms
A
  1. Bence-Jones proteins in the urine

This patient’s presentation is classic for multiple myeloma. Multiple myeloma results in large eosinophilic casts made of Bence-Jones proteins that can be seen on urine microscopy.

Multiple myeloma is cancer of monoclonal plasma cells resulting in the production of large amounts of IgG (55%) or IgA (25%). The amount of light chains (Bence-Jones protein) filtered exceeds the resorptive capacity of the tubule. These proteins precipitate and form large eosinophilic casts, which then obstruct the tubule and interfere with renal function.

148
Q

A 57-year-old male with diabetes mellitus type II presents for a routine check-up. His blood glucose levels have been inconsistently controlled with medications and diet since his diagnosis 3 years ago. At this current visit, urinalysis demonstrates albumin levels of 250 mg/day. All prior urinalyses have shown albumin levels below 20 mg/day. At this point in the progression of the patient’s disease, which of the following is the most likely finding seen on kidney biopsy?

  1. Normal kidney biopsy, no pathological finding is evident at this time
  2. Glomerular hypertrophy with slight glomerular basement membrane thickening
  3. Glomerular basement membrane thickening and mesangial expansion
  4. Kimmelstiel-Wilson nodules and tubulointerstitial fibrosis
  5. Significant global glomerulosclerosis
A
  1. Glomerular basement membrane thickening and mesangial expansion

This patient is suffering from incipient diabetic nephropathy (DN). This stage of DN is characterized by microalbuminuria, which is defined as excretion of 30 to 300 mg/day of albumin. Kidney biopsy findings associated with the incipient stage of diabetic nephropathy include thickening of the glomerular basement membrane and expansion of the mesangium.

149
Q

A 26-year-old male currently undergoing standard therapy for a recently diagnosed active tuberculosis infection develops sudden onset of fever and oliguria. Laboratory evaluations demonstrate high levels of eosinophils in both the blood and urine. Which of the following is most likely responsible for the patient’s symptoms:

  1. Rifampin
  2. Isoniazid
  3. Pyrazinamide
  4. Ethambutol
  5. Return of active tuberculosis symptoms secondary to patient non-compliance with anti-TB regimen
A
  1. Rifampin

This presentation is consistent with acute drug-induced tubulointerstitial nephritis (TIN). Acute drug-induced TIN occurs as a combined type I and type IV hypersensitivity reaction 1-2 weeks after administration of certain drugs: Beta-lactam antibiotics (penicillin), rifampin, sulfonamides, NSAIDs, and diuretics. The rifampin component of the standard anti-TB regimen of isoniazid, rifampin, and pyrazinamide is most likely responsible for the onset of symptoms seen in this patient.

150
Q

A 60-year-old man with a history of osteoarthritis has been awaiting hip replacement surgery for 3 years. During his annual physical, he reports that no amount of analgesic can relieve his constant pain. Laboratory results reveal that his renal function has deteriorated when compared to his last office visit 2 years ago. Serum creatinine is 2.0 mg/dL and urinalysis shows 1+ proteinuria. There are no abnormalities seen on microscopy of the urine. A renal biopsy shows eosinophilic infiltration and diffuse parenchymal inflammation. What is the most likely explanation for this patient’s deterioration in renal function?

  1. Focal segmental glomerulosclerosis
  2. Ischemic acute tubular necrosis
  3. Nephrotoxic acute tubular necrosis
  4. Toxic tubulointerstitial nephritis
  5. Rapidly progressive glomerulonephritis
A
  1. Toxic tubulointerstitial nephritis

The question stem describes a patient who is overusing analgesics. In this scenario, the most likely explanation for deterioration in renal function is NSAID-induced toxic tubulointerstitial nephritis (TIN).

151
Q

A 71-year-old male presents to your office with painless hematuria and left costovertebral angle pain. His body mass index is 33 kg/m^2. A renal biopsy is shown in Figure A. The patient’s mass is most likely a malignancy of which of the following:

  1. Tubular epithelial cells
  2. Perirenal adipose cells
  3. Metastatic adrenocortical carcinoma
  4. Mesangial cells
  5. Visceral epithelial cells
A
  1. Tubular epithelial cells.

The patient presents with classic symptoms of renal cell carcinoma (RCC), including painless hematuria and costovertebral angle pain. Renal cell carcinoma is a malignancy of proximal tubular epithelial cells.

RCC symptoms include painless hematuria, costovertebral angle pain, fever and weight loss. Smoking and obesity are risk factors. Histology shows rounded or polygonal clear cells with abundant clear cytoplasm filled with glycogen and lipid (see image in questions stem). Prognosis is poor, especially if there is involvement of the renal vein.

152
Q

A 65-year-old man with a 40 pack-year history of smoking presents with right costovertebral angle pain, hematuria, fever, and a palpable mass in the right lower quadrant. A chest radiograph reveals multiple nodular masses in the lungs. A biopsy is performed. What is the likely diagnosis?

A

Renal cell carcinoma

153
Q

A 15-month-old male is brought in for a routine exam. His mother reports he is not eating as much as he was previously. On exam the infant is appropriate, alert, and orientated. Abdominal exam indicates a large mass on the right side. The mass is deep and firm and its margins are not palpable. Labs are normal. A biopsy is obtained. What is the likely diagnosis?

A

Wilm’s Tumor

154
Q

A 24 year-old male presented to his physician with a homogenous, painless testicular swelling that he noticed for the past two weeks. After ultrasound and blood tests, he subsequently underwent a radical inguinal orchiectomy and pathology was consistent with seminoma. Which of the following would you expect to see in a tissue sample from the patient’s orchiectomy?

  1. Lobules of large cells with prominent nuclei and watery cytoplasm
  2. Schiller-Duval bodies
  3. Reinke crystals
  4. Tubular papillary architecture with necrosis
  5. Mature derivatives of endoderm, mesoderm, and ectoderm
A
  1. Lobules of large cells with prominent nuclei and watery cytoplasm

Lobules of large cells with prominent nuclei and watery cytoplasm
Seminomas are the most common testicular tumor in males. Though malignant, seminomas have an excellent prognosis if treated early. Seminomas spread first to the para-aortic lymph nodes and then hematogenously. The tumor is treated with orchiectomy followed by radiation if there is suspicion that the disease has spread outside of the testes.

155
Q

A 45-year-old man presents with a 3-day history of right-sided flank pain due to a lodged ureteral stone. What changes would be expected to be seen at the level of glomerular filtration?

  1. Increase in glomerular capillary oncotic pressure
  2. Increase in Bowman’s space capillary oncotic pressure
  3. Increase in Bowman’s space hydrostatic pressure
  4. Increase in filtration fraction
  5. No change in filtration fraction
A
  1. Increase Bowman’s space capillary oncotic pressure

Obstruction of the ureter, as can occur with utereolithiasis, causes an increase in pressure proximal to the obstruction that transmits back to the level of the glomerulus (Bowman’s space) and can lead to hydronephrosis.

156
Q

A 72-year-old male presents to his primary care physician with urinary hesitancy and urinary dribbling that began 6 weeks ago and has gradually worsened. Rectal exam reveals a markedly enlarged prostate. CT scan demonstrates dilated ureters and renal pelvises. Which of the following likely accounts for the CT scan results:

  1. Urethral obstruction
  2. Autosomal dominant polycystic kidney disease
  3. Prerenal azotemia
  4. Elevated serum PSA
  5. Multiple endocrine neoplasia type 1
A
  1. Urethral obstruction

The patient’s clinical scenario suggests urethral obstruction due to prostatic hyperplasia. Prolonged obstruction can lead to hydronephrosis as seen in the patient’s CT scan.

157
Q

A 45-year-old Caucasian male with a history of chronic myeloid leukemia for which he is receiving chemotherapy presents to the emergency room with oliguria and colicky left flank pain. His serum creatinine is 3.0 mg/dL and is urine pH is 5.0. You diagnose nephrolithiasis. His kidney stones, however, are not visible on abdominal x-ray. His stone is most likely composed of which of the following?

  1. Calcium oxalate
  2. Calcium phosphate
  3. Magnesium ammonium phosphate
  4. Uric acid
  5. Cystine
A
  1. Uric acid

Uric acid kidney stones are often associated with leukemia and myeloproliferative disorders.

This patient is most likely suffering from tumor lysis syndrome, in which chemotherapy treatment causes tumor cells to lyse and release uric acid (a metabolite of tumor nucleic acid). Note that uric acid stones require an acidic environment to precipitate, while magnesium ammonium phosphate (struvite) stones require an alkaline environment to precipitate. Uric acid stones also form secondary to gout and renal tubular acidosis.

158
Q

Following passage of a calcium phosphate stone, a 55-year-old male visits his physician to learn about nephrolithiasis prevention. Which of the following changes affecting urine composition within the bladder are most likely to protect against crystal precipitation?

  1. Increased calcium, increased citrate, increased oxalate, increased free water clearance
  2. Decreased calcium, increased citrate, increased oxalate, increased free water clearance
  3. Decreased calcium, decreased citrate, increased oxalate, increased free water clearance
  4. Decreased calcium, increased citrate, decreased oxalate, increased free water clearance
  5. Decreased calcium, increased citrate, increased oxalate, decreased free water clearance
A
  1. Decreased calcium, increased citrate, decreased oxalate, increased free water clearance.

The composition of urine is a critical factor in the process of crystal precipitation and stone formation. Increases in the concentrations of the stone-forming compounds calcium and oxalate can lead to the precipitation of these substances in the urine causing stones to form, while high fluid intake and an associated increase in free water clearance dilutes urine and is protective against stone formation. Urinary citrate binds ionized calcium, preventing stone precipitation and leading to calcium excretion.

159
Q

A 12-year-old boy presents with right flank pain and hematuria. His older brother has had several similar episodes in the past. Results of microscopic urinalysis are shown in Figure A. Which of the following tests can be used to help confirm the underlying diagnosis?

  1. Cyanide-nitroprusside test
  2. Serum uric acid concentration
  3. Urine uric acid concentration
  4. Urine culture
  5. Urease test
A
  1. Cyanide-nitroprusside test.

This boy has the classic presentation of kidney stones, a positive family history, and microscopic analysis showing hexagonal-shaped crystals. These findings are suggestive of cystinuria, which can be confirmed with the cyanide-nitroprusside test.

Kidney stones classically present with abrupt-onset renal colic and severe flank pain, which usually radiates toward the abdomen and groin. Patients often have dysuria and hematuria. Kidney stones are most often seen in adults in their 30’s-40’s. When kidney stones are seen in children, hereditary predisposing factors should be considered. Kidney stones can be of several types, including calcium stones, struvite stones, uric acid stones, and cystine stones.

160
Q

A 49-year-old African American female with a history of chronic myeloid leukemia for which she is receiving chemotherapy presents to the emergency room with oliguria and colicky left flank pain. Her serum creatinine is 3.3 mg/dL. What is the preferred preventative therapy that could have been administered to this patient to prevent her complication of chemotherapy?

  1. Diuresis
  2. Acidification of the urine
  3. Colchicine
  4. Steroids
  5. Dialysis
A
  1. Diuresis.

This patient most likely has acute kidney injury (AKI) and a renal stone secondary to tumor lysis syndrome (TLS). The chief TLS prevention is hydration and diuresis to increase urine flow rate.

161
Q

A female infant is born with a mutation in PKD1 on chromosome 16. An abdominal ultrasound performed shortly after birth would most likely reveal which of the following?

  1. Bilateral kidney enlargement
  2. Microscopic cysts
  3. Horseshoe kidney
  4. Adrenal atrophy
  5. Normal kidneys
A
  1. Normal kidneys

PKD1 is a mutation present in 90% of autosomal dominant polycystic kidney disease (formerly “adult” PKD). In infants with ADPKD, ultrasound will show normal kidneys. Although microscopic cysts are present at birth, they are too small to be detected on abdominal ultrasound.

Autosomal dominant polycystic kidney disease results in large, bilateral cysts that ultimately destroy renal parenchyma in adulthood. This presents with flank pain, hematuria, hypertension, urinary infection, and progressive renal failure. Polycystic liver disease, berry aneurysms, and mitral valve prolapse are associated conditions. Autosomal recessive polycystic kidney disease is a disease with renal cysts that can become apparent in utero and result in renal failure and is associated with Potter’s sequence. Concerns in the neonatal period include hypertension and renal failure.

162
Q

A 35-year-old male with a history of hypertension presents with hematuria and abdominal discomfort. Ultrasound and CT scan reveal large, bilateral cysts in all regions of the kidney. The patient’s disease is most commonly associated with:

  1. Aortic stenosis
  2. Berger’s disease
  3. Diabetes mellitus
  4. Berry aneurysm
  5. Henoch-Schonlein purpura
A
  1. Berry Aneurysm.

The patient’s symptoms and imaging findings are suggestive of adult polycystic kidney disease, or autosomal dominant polycystic kidney disease (ADPKD). ADPKD is associated with Berry aneurysms in the circle of Willis in 10-15% of patients.

ADPCKD usually presents in the 3rd or 4th decade of life. It is marked by multiple, large, bilateral cysts in all regions of the kidney. Cysts may also occur in the liver (30%), pancreas and spleen. Associated complications include berry aneurysm, mitral valve prolapse (MVP), and diverticulosis. Patients are at increased risk for renal cell carcinoma.

163
Q

A 53-year-old man has passed darker urine for the past week. On physical examination there are no abnormal findings. A urinalysis shows pH 5.5, specific gravity 1.013, 2+ blood, no protein, and no glucose. A urine cytology is performed and there are atypical cells seen. A cystoscopy is performed, but no mucosal lesions are noted. He has a 60 pack year history of smoking cigarettes. Which of the following is the most likely diagnosis?

A Adenocarcinoma of prostate

B Urothelial carcinoma of renal pelvis

C Acute interstitial nephritis

D Nodular glomerulosclerosis

E Squamous cell carcinoma of penis

A

B. Urothelial carcinoma of renal pelvis.

The lack of findings in the bladder, but the presence of atypical cells along with hematuria suggests that there is a malignant lesion and it is located higher in the urinary tract. His history of smoking increases the risk for urothelial carcinomas and for renal cell carcinomas.

164
Q

A 62-year-old man has had back pain for the past 8 months. He has had a productive cough for the past 2 days. On physical examination his temperature is 39°C and there is dullness to percussion at the right lung base. Laboratory studies show 4+ gram-positive diplococci in the sputum. A chest radiograph shows right lower lobe consolidation. An abdominal CT scan shows multiple lytic lesions of the vertebrae. On the day prior to death his serum urea nitrogen was 63 mg/dL with creatinine 7.1 mg/dL. A dipstick urinalysis was normal. At autopsy, his kidneys are firm and pale. Microscopically, there is abundant pink hyaline material in glomeruli and around small vessels. This material stains positively with Congo red. Which of the following laboratory findings was most likely to have been present in this patient in the week prior to death?

A Positive antinuclear antibody test

B Serum glucose of 210 mg/dL

C CD4 lymphocyte count of 110/microliter

D Total serum protein of 9.2 g/dL

E Serum prostate specific antigen of 11.8 ng/mL

A

D. Total serum protein of 9.2 g/dL

The findings are consistent with renal amyloid deposition with multiple myeloma. The total protein is elevated from increased serum monoclonal immunoglobulin. Myeloma can produce lytic bone lesions and patients often develop infections with encapsulated bacteria. The excessive light chain production leads to AL amyloid deposition. Bence-Jones proteinuria is often present, but the urine dipstick is most sensitive for albumin, not globulins.

165
Q

what type of kidney stones is more often found in women and almost always result of urinary tract infections?

A

Struvite stones. Organism involved proteus valguaris, kelbseilla are common culprit. they make the urine more alkaline and changes pH and changes uria to ammonia.

166
Q

A 52-year-old previously healthy man has experienced episodes of discomfort with urination for 3 months. There are no remarkable findings on physical examination. Laboratory studies include a urinalysis that reveals 1+ blood. Microscopic urine examination shows numerous RBCs, a few WBCs, and no casts. A urine culture is negative. A plain film radiograph of the pelvis shows a rounded, 1 cm radiopaque lesion in the region of the bladder. Which of the following laboratory test findings is most likely to be present in this man?

A Albuminuria

B Hypercalciuria

C Transaminasemia

D Hemoglobinuria

E Hyperuricemia

A

B. Hypercalciuria

The findings suggest a bladder calculus. Most stones are composed of calcium with oxalate or phosphate. The calcium content makes them radiopaque, unlike pure uric acid stones that are often radiolucent (by conventional x-ray) and which are not very common. The most common cause for urinary tract calculi is idiopathic hypercalciuria.

167
Q

A 72-year-old man has been feeling tired and lethargic for 5 months. He has noted increasing hesitancy with urination. On physical examination his prostate is diffusely enlarged. Laboratory studies show sodium 139 mmol/L, potassium 4.0 mmol/L, chloride 104 mmol/L, CO2 25 mmol/L, creatinine 1.9 mg/dL, and glucose 81 mg/dL. Which of the following renal abnormalities is most likely to be present in this man?

A Cortical atrophy

B Glomerulonephritis

C Papillary necrosis

D Polycystic change

E Renal cell carcinoma

A

A. Cortical atrophy

The prostatic hyperplasia could lead to obstructive uropathy with bilateral hydronephrosis, renal cortical atrophy, and eventual chronic renal failure.

168
Q

A 36-year-old woman has urinary frequency with dysuria for the past 4 days. On physical examination she has no flank pain or tenderness. A urinalysis reveals sp. gr. 1.014, pH 7.5, no glucose, no protein, no blood, nitrite positive, and many WBC’s. She has a serum creatinine of 0.9 mg/dL. Which of the following is the most likely diagnosis?

A Lupus nephritis

B Urinary lithiasis

C Acute cystitis

D Malakoplakia

E Urothelial carcinoma

A

C. Acute cystitis

These are features of acute inflammation. There are no casts, because the infection involves just the bladder, though such an infection could ascend to produce pyelonephritis. Urinary tract infections are more common in women because of the shorter urethra.

169
Q

A 69-year-old man incurs blunt force trauma from a fall. On physical examination he has a contusion on his lower back. An abdominal CT scan shows 3 peripheral 1 to 2 cm cysts in his kidneys. The kidneys are normal in size. Laboratory studies show a serum urea nitrogen of 16 mg/dL and creatinine of 1.1 mg/dL. A urinalysis reveals no blood, ketones, protein, or glucose. Microscopic urinalysis reveals a few oxalate crystals. Which of the following is the most likely diagnosis?

A Polycystic kidney disease

B Hydronephrosis

C Renal atherosclerosis

D Simple cortical cysts

E Recurrent pyelonephritis infection

A

D. Simple cortical cysts

Simple renal cysts typically do not interfere with renal function. A few small cysts can be found in many older persons and are inconsequential. They will appear as incidental findings in radiologic imaging studies. The trauma might cause hemorrhage into a cyst.

170
Q

A clinical study is performed with pediatric subjects who had a diagnosis of minimal change disease. These patients were observed to have prominent periorbital edema at diagnosis. Laboratory test findings from serum and urine tests were analyzed. Which of the following urinalysis test findings is most likely to have been consistently present in these subjects?.

A Nitrite positive

B Proteinuria >3.5 gm/24 hours

C Hematuria with >10 RBC/hpf

D Calcium oxalate crystsls

E Renal tubular epithelial cells and casts

A

B. Proteinuria >3.5 gm/24

This is the definition of nephrotic syndrome. MCD produces a nephrotic syndrome, with significant albuminuria. A single urine specimen will not suffice for the definition of nephrotic syndrome (though it could be extrapolated, given the daunting task of 24 hour urine collection in children).

171
Q

A 12-year-old boy is a member of a family with a history of renal disease, with males more severely affected than females. He is found to have auditory nerve deafness, corneal dystrophy, and ocular lens dislocation. A urinalysis shows microscopic hematuria. A renal biopsy is performed. Microscopically, the glomeruli show glomerular capillaries with irregular basement membrane thickening and attenuation with splitting of the lamina densa. The mesangial matrix is increased and epithelial cells may appear foamy. Which of the following is the most likely diagnosis?

A Goodpasture syndrome

B IgA nephropathy

C Alport syndrome

D Dominant polycystic kidney disease

E Diabetes mellitus, type I

A

C. Alport syndrome

Hereditary nephritis (Alport syndrome) is not associated with immune complexes, but with a genetic defect of type IV collagen production. In most families, it is inherited in an X-linked dominant pattern. Symptoms usually appear at ages 5 to 20, with overt renal failure between ages 20 to 50.

172
Q

A clinical study is performed of laboratory findings in subjects with renal diseases. Loss of physiologic function accompanies many diseases. One renal physiologic function affects thirst. Loss of which of the following renal functions is most likely to be identified by laboratory measurement of the urine specific gravity following water deprivation?

A Filtration

B Reabsorption

C Secretion

D Concentration

E Blood flow

A

D. Concentration

Renal concentrating ability is reflected by the specific gravity. In a person with normal concentrating capacity, less water intake is reflected by an increased specific gravity.

173
Q

A 45-year-old woman has had increasing malaise for the past year. On physical examination her blood pressure is 265/150 mm Hg. Laboratory studies show a plasma renin activity of 9 ng/mL/hr. She then suffers a ‘stroke’ with a right basal ganglia hemorrhage and dies. At autopsy the kidneys are bilaterally small with granular surfaces. Microscopically they show hyperplastic arteriolosclerosis with fibrinoid necrosis, petechial hemorrhages, and microinfarcts in the cortices. Which of the following conditions is most likely to be her underlying cause of death?

A Diabetes mellitus, type II

B Fibromuscular dysplasia

C Factor V Leiden mutation

D Analgesic abuse

E Systemic sclerosis

A

E. systemic sclerosis

She has findings of hyperplastic arteriolosclerosis with severe, malignant hypertension. Renal disease is likely to complicate diffuse scleroderma, but not the more limited form of scleroderma sometimes called ‘CREST’ syndrome.

174
Q

A 3-year-old child has become more irritable over the past two months and does not want to eat much at meals. On physical examination the pediatrician notes an enlarged abdomen and can palpate a mass on the right. An abdominal CT scan reveals a 10 cm solid mass involving the right kidney. The resected mass has a microscopic appearance with sheets of small blue cells along with primitive tubular structures. The child receives chemotherapy and radiation therapy, and there is no recurrence. Which of the following neoplasms is this child most likely to have had?

A Angiomyolipoma

B Renal cell carcinoma

C Urothelial carcinoma

D Wilms tumor

E Medullary fibroma

A

D. Wilm’s tumor

This is the classic age, histopathology, and location for Wilms tumor. These childhood neoplasms, when treated properly, have a very good prognosis.

175
Q

A 5-year-old boy is noted to have increased puffiness around his eyes for the past week, and he has been less active than normal. On physical examination he has periorbital edema. Vital signs include T 37°C, P 75/minute, RR 22/minute, and BP 140/90 mm Hg. A urinalysis reveals sp. gr. 1.010, pH 6.5, no glucose, 4+ protein, no blood, no casts, and no ketones. Microscopic urinalysis reveals oval fat bodies, but no WBC’s or RBC’s. He improves following a course of corticosteroid therapy. Which of the following renal lesions is most likely to have been present in this boy?

A Glomerular crescent formation

B Podocyte foot process effacement

C Patchy acute tubular necrosis

D Hyperplastic arteriolosclerosis

E Mesangial immune complex deposition

A

B. Podocyte foot process effacement

This is minimal change disease, the most common cause for nephrotic syndrome in children, and fusion of podocyte foot processes is the only pathologic finding present (on electron microscopy). Most patients respond to corticosteroid therapy.

176
Q

A clinical study is performed involving subjects with glomerulonephritis. One group of subjects has a diagnosis of crescentic glomerulonephritis and another group has membranous nephropathy. Which one of the following laboratory findings is most likely to be found in the absence of other findings in subjects with membranous nephropathy?

A Rapid onset

B Red blood cell casts

C Oliguria

D Albuminuria

E Hypertension

A

D. Albuminuria

Proteinuria may be present with nephritic syndromes (characterized by RBCs spilled into the urine from the glomeruli), but it is usually not marked, as in nephrotic syndrome. Membranous nephropathy typically produces nephrotic syndrome that is predominantly albuminuria. Diseases with crescents, such as membranoproliferative glomerulonephritis, lead to nephritic syndrome.

177
Q

A 51-year-old man is hospitalized for acute myocardial infarction. He has decreased cardiac output with hypotension requiring multiple pressor agents. His urine output drops over the next 3 days. His serum urea nitrogen increases to 59 mg/dL, with creatinine of 2.9 mg/dL. Urinalysis reveals no protein or glucose, a trace blood, and numerous hyaline casts. Five days later, he develops polyuria and his serum urea nitrogen declines. Which of the following pathologic findings in his kidneys is most likely to have caused his azotemia?

A Patchy tubular necrosis

B Podocyte foot process effacement

C Glomerular crescent formation

D Hyperplastic arteriolosclerosis

E Mesangial immune complex deposition

A

A. Patchy tubular necrosis

He has findings of ischemic acute tubular necrosis (ATN) from heart failure with hypotension. A clue is the >20:1 ratio of urea nitrogen to creatinine, which occurs early in the course, from prerenal azotemia. As the disease progresses, the ratio begins to approach 10:1, typical for intrinsic renal diseases. ATN may also be produced by toxins such as ethylene glycol in antifreeze, but the tubular necrosis is more diffuse.

178
Q

A clinical study is performed to determine the value of percutaneous renal biopsy. The medical records of subjects with renal diseases are analyzed to note the circumstances in which the results of a renal biopsy facilitated choice of therapy that improved prognosis. In which of the following situations is a percutaneous needle biopsy of the kidney most useful?

A Fever and flank pain with suspected acute pyelonephritis

B Prostatic hyperplasia with suspected hydronephrosis

C Premature neonate with suspected polycystic kidney disease

D Abdominal pain with suspected renal cyst

E Acute renal failure with suspected systemic lupus erythematosus

A

E. acute renal failure with suspected SLE

Therapy may depend upon determination of the severity and nature of the renal disease with SLE.

179
Q

A 56-year-old man complains of dull flank pain for the past month. On physical examination he has tenderness to percussion at the right costovertebral angle. Laboratory studies show microscopic hematuria but no proteinuria or glucosuria. A urine cytology shows no atypical cells. A CBC shows WBC count 7800/microliter, Hgb 21.1 g/dL, Hct 63.5%, MCV 94 fL, and platelet count 195,000/microliter. His serum urea nitrogen is 15 mg/dL and creatinine 1 mg/dL. Which of the following radiographic findings is most likely to be present in this man?

A Hydronephrosis on intravenous pyelogram

B Renal mass on abdominal CT scan

C Radiopaque ureteral calculus on an abdominal plain film

D Enlarged, multicystic kidneys on abdominal ultrasound

E Pelvic abscess below the bladder on MR imaging

A

B. Renal mass on abdominal CT scan

The polycythemia suggests a paraneoplastic syndrome, and a renal cell carcinoma is a likely candidate for the primary lesion. The flank pain and hematuria can be explained by the mass effect from a renal cell carcinoma.

180
Q

A 43-year-old man goes to his physician for a routine check of his health status. He is found to have a blood pressure of 150/95 mm Hg. His urinalysis shows pH 6.5, specific gravity 1.015, no glucose, blood, or protein, and no casts. His serum creatinine is 1.4 mg/dL. If he is not treated, which of the following conditions will most likely cause his death?

A Intracerebral hemorrhage (stroke)

B Aortic aneurysm rupture

C Congestive heart failure

D Chronic renal failure

E Intracranial aneurysm rupture

A

C. Congestive heart failure

Hypertension leads to cardiac enlargement, then dilation, and eventual heart failure. This is the most common outcome with untreated hypertension.

181
Q

A 20-year-old previously healthy man has been feeling tired for the past 5 days. He then passes dark-colored urine. On physical examination his blood pressure is 160/90 mm Hg. Laboratory studies show his serum creatinine is 4.4 mg/dL and BUN 40 mg/dL. A urinalysis reveals pH 6, specific gravity 1.011, 3+ blood, 1+ protein, no glucose, and no ketones. On urine microscopic examination there are numerous RBC casts. Which of the following pathologic findings on renal biopsy is most likely to be present in this man?

A Glomerular crescents

B Widened proximal tubules

C Neutrophilic infiltrates

D Basememt membrane thickening

E IgA deposited in glomerular capillaries

A

A. Glomerular crescents

182
Q

A 43-year-old man has had increasing malaise for the past 3 weeks. On physical examination he has a blood pressure of 150/95 mm Hg and 1+ pitting edema of the lower extremities to the knees. Dipstick urinalysis shows no glucose, blood, ketones, nitrite, or urobilinogen, and the microscopic urinalysis reveals no RBC/hpf and only 1 WBC/hpf. Additional laboratory studies show a 24 hour urine protein of 4.1 gm. His serum creatinine is 3.5 mg/dL with urea nitrogen of 38 mg/dL. His hepatitis B surface antigen is positive. Which of the following is the most likely diagnosis?

A Membranous nephropathy

B Systemic lupus erythematosus

C Acute tubular necrosis

D Diabetic nephropathy

E Post-streptococcal glomerulonephritis

A

A. Membranous nephropathy

Membranous nephropathy is the most common cause for nephrotic syndrome in adults. Some cases are associated with underlying infections or malignancies, but in most cases the cause is unknown. There is diffuse thickening of the glomerular capillary basement membrane from immune deposits.

183
Q

A 60-year-old woman is admitted with sudden onset of chest pain and is diagnosed with an acute myocardial infarction. There is difficulty maintaining adequate blood pressure and tissue perfusion for 3 days. Her serum lactate becomes elevated. Her serum urea nitrogen increases to 44 mg/dL and creatinine to 2.2 mg/dL. Granular and hyaline casts are present on microscopic urinalysis. Which of the following renal lesions is most likely to be present in this situation?

A Chronic pyelonephritis

B Acute tubular necrosis

C Nodular glomerulosclerosis

D Renal vein thrombosis

E Minimal change disease

A

A. Acute tubular necrosis

Ischemia, typically in hypotensive hospitalized patients, is the most frequent antecedent to acute tubular necrosis (ATN), and ischemic heasrt disease with coronary syndromes including MI’s are common. This is a pre-renal form of azotemia, and note the BUN:Cr ratio more than 20:1.

184
Q

A 50-year-old man was diagnosed at age 15 with type 1 diabetes mellitus. His disease has been poorly controlled, as evidenced by elevated hemoglobin A1C levels. He develops a non-healing ulcer of his foot at age 35. At age 45, he has an increasing serum urea nitrogen and a urinalysis shows sp gr 1.012, pH 6.5, 1+ protein, no blood, 1+ glucose, negative leukocyte esterase, negative nitrite, and no ketones. Which of the following renal diseases is he most likely to have?

A Nodular glomerulosclerosis

B Hyperplastic arteriolosclerosis

C Papillary necrosis

D Crescentic glomerulonephritis

E Pyelonephritis

A

A. Nodular glomerulosclerosis

Nodular glomerulosclerosis is a typical complication of long-standing diabetes mellitus. Microalbuminuria may preceed development of other abnormalities.

185
Q

A 39-year-old woman is found to have a blood pressure of 160/105 mm Hg while at a free health screening clinic. She feels fine and has had no major medical problems in her life. An abdominal ultrasound reveals that the left kidney is smaller than the right, but that neither is cystic and no masses appear to be present. MR angiography reveals focal narrowing with thickening and beading of the left main renal artery. A urinalysis reveals no abnormal findings. She has an elevated plasma renin. Which of the following is the most likely diagnosis?

A Diabetes mellitus

B Antiphospholipid syndrome

C Fibromuscular dysplasia

D Thrombotic thrombocytopenic purpura

E Cholesterol emboli syndrome

A

C. FIbromuscular dysplasia
Fibromuscular dysplasia is an uncommon but surgically treatable cause for hypertension. The abnormal segment of artery can be treated with angioplasty or removed and replaced with a graft. She has a ‘Goldblatt’ kidney from narrowing of one renal artery, leading to an increase in renin, with hypertension, and the subsequent hypertensive changes can eventually damage the opposite kidney.

186
Q

A 70-year-old woman has had a fever for the past 3 days. She has burning dysuria. On physical examination her temperature is 37.8°C and there is dull pain on palpation of her left lower back. Laboratory studies show Hgb 13.3 g/dL, Hct 40.2%, and WBC count 12,300/microliter with differential count 72 segs, 9 bands, 13 lymphs, 5 monos, and 1 eosinophil. A urine dipstick analysis shows sp gr. 1.017, pH 6, leukocyte esterase positive, nitrite positive, protein negative, glucose negative, and blood negative. Which of the following microscopic urinalysis findings would be most diagnostic for her renal disease?

A Dysmorphic red blood cells

B Oval fat bodies

C Renal tubular epithelial cells

D White blood cell casts

E Triple phosphate crystals

A

D. WBC cast

The WBC casts are most characteristic for an acute interstitital nephritis (acute pyelonephritis). The casts can only come from distal tubules / collecting ducts. The positive leukocyte esterase and nitrite are typical for acute inflammation with bacterial infection of the urinary tract.

187
Q

A 53-year-old woman has had chronic arthritis pain for the past 3 years. She has taken 2 gm of phenacetin and acetaminophen a day for her pain over that time. She now has increasing fatigue. There are no abnormal findings on physical examination. Laboratory studies show her serum urea nitrogen is 52 mg/dL and creatinine 5.4 mg/dL. Which of the following pathologic findings is most likely to occur in her kidneys?

A Papillary necrosis

B Focal segmental glomerulosclerosis

C Nephrocalcinosis

D Acute interstitial nephritis

E Arteriolosclerosis

A

A. Papillary necrosis

She has analgesic abuse nephropathy which leads to papillary necrosis and chronic interstitial nephritis with tubular atrophy (though the renal columns are spared). There is also risk for development of urothelial carcinoma. The analgesics implicated may include combinations of phenacetin, acetaminophen, and aspirin.

188
Q

A 25-year-old woman has been hospitalized for treatment of a Staphylococcus aureus abscess of her left thigh complicating a puncture wound. The wound is incised and drained and she receives antibiotic therapy. She is improving and discharged home a week later, but the next day she develops a fever. On physical examination her temperature is 38.1°C and there is a diffuse erythematous skin rash of her trunk and extremities. A urinalysis shows sp gr 1.020, pH 6.5, 1+ blood, 1+ protein, no glucose, and no ketones. There are 10-20 WBCs/hpf and 1-5 RBCs/hpf, and a few eosinophils are noted on urine microscopic examination. Which of the following is the most likely diagnosis?

A Acute tubular necrosis

B Septicemia with pyelonephritis

C Drug-induced interstitial nephritis

D Hemolytic-uremic syndrome

E Post-infectious glomerulonephritis

F Urinary tract infection

A

C. Drug-induced interstitial nephritis

This allergic response can occur following drug therapy with such antibiotic agents as methicillin, as well as some diuretics and NSAIDs. This allergic response with acute interstitial nephritis may be unrelated to the amount of drug and duration of therapy. This condition is treated by stopping the drug.

189
Q

A 39-year-old previously healthy man has the sudden onset of severe right flank pain that comes in waves all night long. When he is seen in the emergency room, after waiting for two hours, he is exhausted. On physical examination there are no abnormal findings. Urinalysis reveals no ketones, glucose, protein, nitrite, or urobilinogen, but blood is present. Urine microscopic examination shows many RBCs but few WBCs. The specific gravity is 1.015 and the pH is 5.5. Which of the following is the most likely diagnosis?

A Nodular prostatic hyperplasia

B Membranous nephropathy

C Ureteral calculus

D Renal angiomyolipoma

E Urothelial carcinoma of bladder

A

C. Ureteral calculus

These acute symptoms are typical for a calculus that is being passed down the ureter, with intermittent contractions producing colicky pain. 5 to 10% of persons may pass a urinary stone at some point in their life. Non-contrast CT scanning is a good diagnostic study. Most calculi will pass spontaneously, but analgesics are needed. NSAIDS work well except when renal function is impaired; opiates may be considered. Other patients waiting in the ER would gladly let this man be seen first, and the ER staff should triage patients in acute distress first.

190
Q

A 15-year-old girl has had increasing lethargy following a bout of the ‘flu’ 3 weeks ago. On physical examination there are no abnormal findings. Her condition does not improve after 3 weeks on corticosteroid therapy, so a renal biopsy is performed and microscopic examination shows segmental sclerosis of 3 of 10 glomeruli. Immunofluorescence studies and electron microscopy do not show immune deposits. Which of the following is the most likely outcome for this girl’s condition?

A Progression to chronic renal failure

B Improvement with additional corticosteroid therapy

C Development of restrictive lung disease

D Discovery of an underlying malignancy

E Remission following dietary change

A

A. Progression to chronic renal failure

The findings point to focal segmental glomerulosclerosis (FSGS), which leads to chronic renal failure in half of cases. The lack of resolution with corticosteroid therapy and the progression to chronic renal failure is what sets FSGS apart from minimal change disease. FSGS is idiopathic, so there is unlikely to be an underying condition.

191
Q

A 59-year-old man notes blood in his urine for the past week. On physical examination there are no abnormal findings. A urinalysis confirms the presence of blood, but no proteinuria or glucosuria. A urine culture is negative. A cystoscopy is performed, and a 3 cm exophytic mass is seen in the dome of the bladder. A biopsy of this mass is performed and microscopic examination reveals fibrovascular cores covered by a thick layer of transitional cells. Which of the following risk factors is most likely to have led to development of this lesion?

A Diabetes mellitus

B Recurrent urinary tract infection

C Therapy with methicillin

D Cigarette smoking

E Tuberous sclerosis

F Use of NSAIDS

A

D. Cig smoking

He has a urothelial carcinoma of the urinary bladder, and smokers are at increased risk for this cancer. These cancers can be multiple and recurrent. Additional less common risk factors include exposure to aniline dyes and to beta-naphthylamine compounds. Drugs that increase the risk include analgesic phenacetin and the chemotherapy agent cyclophosphamide.

192
Q

A 53-year-old woman has noted fever and right-sided flank pain for the past 3 days. On physical examination her temperature is 38.4°C and there is right costovertebral angle tenderness. A urinalysis reveals sp. gr. 1.010, pH 7.5, no glucose, no protein, no ketones, and 1+ blood. Many WBCs and WBC casts are seen on urine microscopic examination. An abdominal radiograph reveals a radiopaque calculus that forms a cast of a dilated right renal collecting system. A urine culture grows Proteus vulgaris. Which of the following crystals is most likely to be seen in large numbers on microscopic urinalysis in this woman?

A Calcium oxalate

B Cystine

C Calcium phosphate dihydrate

D Uric acid

E Magnesium ammonium phosphate

A

E. Magnesium ammonium phosphate

She has a staghorn calculus and acute pyelonephritis. These ‘infection stones’ are typically the ‘triple phosphate’ stones whose formation is aided by infection with urea-splitting bacteria such as Proteus.

193
Q

A 60-year-old man was diagnosed last year with adenocarcinoma of the lung, and he underwent right lower lobectomy. For the past 3 weeks he has had increasing malaise. On physical examination he has pitting edema to his knees and presacral edema. Abdominal and chest CT scans show scattered hepatic mass lesions and hilar lymphadenopathy. A urinalysis reveals 4+ proteinuria, and his 24 hour urine protein is 2.7 gm. His serum urea nitrogen is 55 mg/dL with creatinine of 6.1 mg/dL. A renal biopsy is performed, and there is focal deposition of IgG and C3 with a granular pattern. Which of the following forms of glomerular disease is he most likely to have?

A Membranous nephropathy

B Rapidly progressive glomerulonephritis

C Nodular glomerulosclerosis

D Chronic glomerulonephritis

E Dense deposit disease

A

A. Membranous nephropathy

Most cases of membranous nephropathy are idiopathic, but in some patients there is a history of an infection or a malignancy (usually lung) with antigenemia that drives the immune deposition in the glomerular capillary basement membranes. Membranous nephropathy produces primarily proteinuria. Note the BUN:Cr ratio here close to 10:1 that suggests intrinsic renal disease.

194
Q

A clinical study is performed with subjects born with congenital urinary tract anomalies. Their records are reviewed to assess the development of long term complications. One group of subjects is found to have an increased risk for both infection and development of a carcinoma. Which of the following congenital urinary tract anomalies is most likely to carry this risk?

A Unilateral renal agenesis

B Bladder exstrophy

C Bilateral ureteral duplication

D Horseshoe kidney

E Medullary sponge kidney

A

B. Bladder exstrophy

Bladder exstrophy is a serious condition, and requires surgical repair. The exposed bladder predisposes to infection. There is a long-term risk for carcinoma, such as bladder adenocarcinoma or colonic adenocarcinoma.

195
Q

A 57-year-old man has had dysuria for the past week. Over the past 2 days he has experienced shaking chills. On physical examination his temperature is 39.3°C. A urinalysis shows sp gr 1.016, pH 6, 1+ glucose, 1+ blood, no ketones, and no protein. Urine microscopic examination shows numerous WBCs and WBC casts. His serum creatinine is 1.5 mg/dL and glucose 155 mg/dL with hemoglobin A1C 8.7%. A renal ultrasound scan shows a 0.3 cm free floating echodense object in the left renal pelvis. Which of the following complications has this man most likely developed?

A Acute tubular necrosis

B Aspergillus fungus ball

C Cystine-containing calculus

D Hematoma formation

E Papillary necrosis

F Renal cell carcinoma

G Urothelial carcinoma

A

E. Papillary necrosis

A calculus may form as a consequence of infection, but such stones are typically formed of magnesium ammonium phosphate.

196
Q

A 48-year-old woman has had increasing malaise for the past 6 months. On physical examination there are no abnormal findings except for diminished sensation to pinprick and light touch in her lower legs and feet. She is afebrile and normotensive. Laboratory studies show serum creatinine 4.5 mg/dL, urea nitrogen 42 mg/dL, glucose 130 mg/dL, and hemoglobin A1C 7.9%. A urinalysis shows 1+ glucose, 1+ protein, no blood, and no ketones. Urine microscopic examination shows 1 RBC/hpf and 1 WBC/hpf. Which of the following pathologic abnormalities is she most likely to have in her kidneys?

A Acute pyelonephritis

B Acute tubular necrosis

C Chronic glomerulonephritis

D Hydronephrosis

E Hyperplastic arteriolosclerosis

F Membranous nephropathy

G Nodular glomerulosclerosis

H Polycystic change

A

G. Nodular glomerulosclerosis

The classic lesion with diabetes mellitus is nodular glomerulosclerosis, which gradually reduces renal function. Diffuse glomerulosclerosis may also be present. Microscopic albuminuria may be the clue that diabetic nephropathy is developing. The glucosuria suggests poor control of glycemia, confirmed by the elevated Hgb A1c.

197
Q

Question 35
A 49-year-old woman has been hospitalized for the past 10 days for treatment of bronchopneumonia. She has developed chills and fever over the past 2 days. On physical examination her temperature is 38.8°C and she has a diffuse erythematous skin rash. Laboratory studies show serum creatinine 2.2 mg/dL and glucose 73 mg/dL. A peripheral blood smear reveals eosinophilia. On urinalysis she has 2+ proteinuria but no blood, glucose, or ketones. Which of the following is the most likely diagnosis?

A Post-streptococcal glomerulonephritis

B Drug-induced interstitial nephritis

C IgA nephropathy

D Acute tubular necrosis

E Acute serum sickness

A

B. Drug-induced interstitial nephritis

These findings are typical for a drug-induced acute interstitial nephritis. The eosinophilia is seen with allergic phenomena (as in a drug allergy).

198
Q

A 59-year-old man has experienced lower back pain for 4 months. On physical examination there are no abnormal findings. A urinalysis shows microscopic hematuria, but no proteinuria or glucosuria. An abdominal CT scan reveals a 6 cm solid mass in the upper pole of the right kidney. A right nephrectomy is performed, and the grossly variegated mass is seen microscopically to be composed of nests of cells with clear cytoplasm. Which of the following laboratory test findings likely to be associated with this lesion?

A Hypercalcemia

B Increased catecholamines

C Positive serology for hepatitis B surface antigen

D Hyponatremia

E Hyperaldosteronemia

A

A. Hypercalcemia

This paraneoplastic effect can occur with renal cell carcinomas. The hypercalcemia is most likely related to elaboration of parathormone-related peptide (PTHrP) by the neoplasm.

199
Q

A 30-year-old man has noted puffiness around his eyes and swelling of his feet for the past 2 weeks. On physical examination his blood pressure is 155/95 mm Hg. Urine microscopic examination reveals oval fat bodies. Which of the following conditions is he most likely to have?

A Ascending pyelonephritis

B Nephritic syndrome

C Nephrotic syndrome

D Obstructive uropathy

E Renal infarction

F Papillary necrosis

A

C. Nephrotic syndrome

Oval fat bodies appear in association with pronounced proteinuria and lipiduria. These bodies represent degenerating, sloughed tubular cells that are filled with lipid. The tubular cells try to reabsorb the spilled protein and lipid and get overwhelmed.

200
Q

Question 42
A 55-year-old man is found down and unconscious. On physical examination he is afebrile. After catheterization, he passes a small amount of dark urine. The urine dipstick test for blood is positive but no red blood cells are seen on microscopic examination of the urine sediment. Which of the following is the most likely diagnosis?

A Post-streptococcal glomerulonephritis

B Renal papillary necrosis

C Ureteral lithiasis

D Rhabdomyolysis

E Renal infarction

A

D. Rhabdomyolysis

The dipstick test is sensitive for both hemoglobin and myoglobin. Myoglobinuria may occur with rhabdomyolysis from muscle injury. Trauma, including prolonged immobilization in one position, may cause muscle injury.

201
Q

Epithelial crescents is associated with what nephritic syndrome?

A

RPGN

202
Q

RPGN is divided into three groups based on IF findings. Indicate which of these antibodies is associated with which type.
A. Anti-glomerular basement membrane disease
B. Immune complex disease
C. absence of anti-GBM antibodies and immune complex

A

A. Type I (Goodpasture syndrome – needs plasmapheresis)
B. Type II (associated with Lupus, Henoch-schonlein purpura)
C. Type III (includes granulomatosis iwth polyangiitis (ANCA-assoicated vasculitis) and microscopic polyangiitis.

203
Q

Necrotizing papilitis with papillary necrosis is a complication of _

A

acute pyelonephritis and diabetics are particular prone to this development.

204
Q
Most common cause of UTI in older women is e.coli and often presents with WBC casts.  Seeing WBC casts in urine sample indicates the infection has spread to 
A. urethra
B. Bladder
C. Cervix
D. kidney
E. Ureters
A

D. Kidney.

Casts are formed in the renal tubules.

205
Q

Presence of Bence Jones proteinuria is a characteristic of what disease?

A

Multiple Myeloma

206
Q

A 63-year-old man has noted increasing back pain for 7 months. He has had three respiratory tract infections with Streptococcus pneumoniae during the past year. On examination, he has pitting edema to his thighs. Laboratory studies show total serum protein, 9.6 g/dL; albumin, 3.5 g/dL; creatinine, 3 mg/dL; urea nitrogen, 28 mg/dL; and glucose, 79 mg/dL. Urinalysis shows proteinuria of 4 g/24 hr, but no glucosuria or hematuria. Abdominal CT scan shows enlarged kidneys without cysts or masses. A renal biopsy specimen stained
with H&E shows deposits of amorphous pink material within glomeruli, interstitium, and arteries. Which of the following diseases is he most likely to have?

A Analgesic nephropathy

B ANCA-associated granulomatous vasculitis

C Type 2 diabetes mellitus

D Membranous nephropathy

E Multiple myeloma

F Systemic lupus erythematosus

A

E. MM.

There is a large amount of serum globulin, back pain from lytic lesions, immunosuppression with recurrent in- fections, and amyloid deposition enlarging the kidneys, all consistent with multiple myeloma. This AL amyloid deposi- tion occurs in 6% to 24% of myeloma cases, and can involve kidneys. Patients with myeloma often have Bence Jones pro- teinuria (not detected by the standard dipstick urinalysis), and some have cast nephropathy, which can cause acute or, more commonly, chronic renal failure.